Download as pdf or txt
Download as pdf or txt
You are on page 1of 181

UNIT 1

NUMBERS(
A number is a mathematical object used to count and measure. A notational
symbol that represents a number is called a numeral but in common use, the word
number can mean the abstract object, the symbol, or the word for the number.

Number is first discovered in India and ten symbols are used, namely o, 1, 2, 3, 4,
5, 6, 7, 8 and 9 to represent any number. These symbols are called digits.

A group of digits, denoting a number, is called a numeral. Every digit in a numeral


has two values:

a. Its face value is equal to it and never changes.


b. Its place value depend on its position in numeral.

1. Natural Number : N = {1, 2, 3, 4, 5, 6, 7, 8, 9, …… ∞}$(0$excluding)


2. Whole Numbers : W= {0, 1, 2, 3, 4, 5, 6, 7, 8, 9, …… ∞} (0 including) .
All natural numbers are whole numbers. 0 is only a whole number and not a
natural number. Aaryabhatt, who was an astronomer and lived in India around
500 AD, invented the symbol 0 for nothing.
3. Even Number : E= {2, 4, 6, 8, 10 ………………..}
4. Odd Numbers : O= {1, 3, 5, 7, 9, 11, ……}
5. Positive integers : I+ = {+1, +2, +3, …….}
6. Negative Integers : I- = {-1, -2, -3, -4,…………..$}
7. Integers : I= {…24,23,22,21,0,1,2,3,4…..}
8. Zero :
a. Zero is an integer and it is neither positive nor negative.
b. Any number multiplied by 0 is equal to 0.
c. Face value and place value of 0 is always 0
9. Imaginary Numbers: The square root of negative numbers are called
imaginary numbers. Example : , , etc.
10. Real Numbers : When we put the irrational numbers together with the rational
numbers, we finally have the complete set of real numbers. Real numbers are
• Rationals + Irrationals
• All points on the number line
• Or all possible distances on the number line

Real numbers include

1
• Whole Numbers (like 1,2,3,4,5,6 etc)
• Rational Numbers (like 3/4, 0.1, 0.635..., 1.5, etc )
• Irrational Numbers (like π, √3, etc )

What is not a Real number ?

• √-4 (the square root of minus 4) is not a Real Number, it is an


Imaginary Number
• Infinity is not a Real Number

The Real Number Line

The Real Number Line is like an actual geometric line.

A point is chosen on the line to be the "origin", points to the right will be positive,
and points to the left will be negative.

A distance is chosen to be "1", and the whole numbers can then be marked off:
{1,2,3,...), and also in the negative direction: {-1,-2,-3, ...}

Any point on the line is a Real Number:

2
• The numbers could be rational (like 20/9)
• or irrational (like$!)

Properties of Real Number :

a. Closure Property : The sum and multiplication of the real numbers


are always a real number. Example : x + y = z and a x b = c, here x,
y, z, a, b and c are integers.
b. Opposite : Two real numbers that are the same
distance from the origin of the real number line are opposites of each
other.
c. Reciprocals
reciprocals of each other.
!
: Two numbers whose product is 1 are

d. Commutative Property : When adding or multiplying two numbers,


the order of the numbers does not matter. Example : a + b = b + a or
a x b = b x a, where a and b are rational numbers.
e. Associative Property : When three numbers are added, it makes
no difference which two numbers are added first. And at the same
time when three numbers are multiplied, it makes no difference which
two numbers are multiplied first. Example : (a+b)+c=a+(b+c) or
(axb) x c= ax (bxc), where a, b and c are rational numbers.
f. Distributive Property : Multiplication distributes over addition ie.

!
ax(b+c)=(axb) + (axc), where a, b and c are integers.
g. Additive Identity Property : The additive identity property states that if
0 is added to a number, the result is that number. Example :
5+0=0+5=5
h. Multiplicative Identity Property: The multiplicative identity property
states that if a number is multiplied by 1, the result is that number.
Example : 5 x 0= 0 x 5=0
i. Additive Inverse : The additive inverse property states that
opposites add to zero ie. if the sum of two rational numbers is zero the
numbers are known as additive inverse of the other. Example :
+ ( )=0. Additive inverse of every rational number x is –x.
j. Multiplication Inverse : The multiplicative inverse property states
that reciprocals multiply to 1 ie. if the multiplication of two numbers is
1, then each number is known as multiplication inverse of the other.
Example : 3 x = 1 or x = 1. Multiplication inverse of every

rational number x will be


11. Composite Numbers : A composite number is a positive integer which has a
positive divisor other than one or itself. In other way a Composite number has
more than two divisors. i.e. 10 is a composite number as it has 1,2,5 and 10
as divisors. Other examples : 4,6,9,16,21,… etc.

When a number can be divided up evenly it is a Composite Number

3
12. Prime Numbers :

When a number can not be divided up evenly it is a Prime Number.

A natural number other than 1, which has no other factor except 1 and itself,
is called a Prime Number. Examples : 2,3,5,7,11,13,17,…… are prime numbers.

When only two factors of a number are 1 and the number, then the number is
called Prime Number and it must be greater than 1.

A prime number is greater than 1 and a Positive integer.

Any natural number greater than 1 is either composite or prime number.

List of prime number :

2 3 5 7 11 13 17 19 23 29 31 37 41 43 47 53 59 61 67
71 73 79 83 89 97 101 103 107 109 113 127 131 137 139 149 151 157 163
167 173 179 181 191 193 197 199 211 223 227 229 233 239 241 251 257 263 269
271 277 281 283 293 307 311 313 317 331 337 347 349 353 359 367 373 379 383
389 397 401 409 419 421 431 433 439 443 449 457 461 463 467 479 487 491 499
503 509 521 523 541 547 557 563 569 571 577 587 593 599 601 607 613 617 619
631 641 643 647 653 659 661 673 677 683 691 701 709 719 727 733 739 743 751
757 761 769 773 787 797 809 811 821 823 827 829 839 853 857 859 863 877 881
883 887 907 911 919 929 937 941 947 953 967 971 977 983 991 997

1 is neither Prime nor Composite number.

The smallest prime number is 2.

2 is the only even Prime number, all other Prime Numbers are odd.

13.Co-Prime Numbers : Two numbers are said to be Coprimes when they have
only 1 as a common factor.
Two natural numbers are called co-prime numbers if they have no common
factor other than 1 or two natural numbers are co-prime if their H.C.F is 1.

Example 1 : Find out whether number 7 and 12 are Coprime ?

4
Answer :

Factors of 7 are 1 and 7


Factors of 12 are 1, 2,3,4 and 12
On Comparing the factors of number 7 and 12, we find that both have only 1
as a common factor. Hence, the numbers 7 and 12 are Coprimes.

14.Twin Prime Number : The pairs of Prime Numbers whose difference is 2 are
called Twin Prime Numbers. Example : (5,7), (11,13),(17,19) etc.
15.Prime Triplet Number : The three natural numbers whose H.C.F. is 1, are
called Prime triplet Numbers. Example; (8,9,25).

In mathematics, a prime triplet is a set of three prime numbers of the form


(p, p + 2, p + 6) or (p, p + 4, p + 6). With the exceptions of (2, 3, 5) and (3,
5, 7), this is the closest possible grouping of three prime numbers, since every
third odd number greater than 3 is divisible by 3, and hence not prime.

The first prime triplets are

(5, 7, 11), (7, 11, 13), (11, 13, 17), (13, 17, 19), (17, 19, 23), (37, 41, 43),
(41, 43, 47), (67, 71, 73), (97, 101, 103), (101, 103, 107), (103, 107, 109),
(107, 109, 113), (191, 193, 197), (193, 197, 199), (223, 227, 229), (227,
229, 233), (277, 281, 283), (307, 311, 313), (311, 313, 317), (347, 349,
353), (457, 461, 463), (461, 463, 467), (613, 617, 619), (641, 643, 647),
(821, 823, 827), (823, 827, 829), (853, 857, 859), (857, 859, 863), (877,
881, 883), (881, 883, 887)

A prime triplet contains a pair of twin primes (p and p + 2, or p + 4 and p + 6


), a pair of cousin primes (p and p + 4, or p + 2 and p + 6), and a pair of sexy
primes (p and p + 6).

16. Perfect number : A given number for which sum of all its factors is equal
to it's twice, the number is known as A Perfect Number. Example :

Examples 1 = Is 6 a Perfect number ?


Answer = Find the factors of 6
Factors of 6 = 1, 2, 3, 6.
Add all the above factors.
Sum = 1 + 2 + 3 + 6 = 12
Twice of given number 6 = 12
Since the sum of all the factors of 6 is equal to its twice
So, 6 is a perfect number.

Examples 2 = Is 9 a Perfect number ?


Answer = Find the factors of 9.

5
Factors of 9 = 1, 3, 9.
Add all the above factors.
Sum = 1 + 3 + 9 = 13
Twice of given number 9 = 18
Since the sum of the factors of 9 is not equal to its twice
Hence, 9 is not a perfect number.

Examples 3 = Is 28 a Perfect number ?


Answer = Find the factors of 28
Factors of 28 = 1, 2, 4, 7, 14, 28.
Add all the above factors.
Sum = 1 + 2 + 4 + 7 + 14 + 28 = 56
Twice of given number 28 = 56
Since, the sum of all the factors of 28 is equal to its twice
So, 28 is a perfect number.

17.Literal Numbers : An English Alphabet that is used to represent a variable


is called a Literal Number.
18.Fraction : A fraction is a quantity that can not be expressed in
whole number. A fraction is an ordered number of whole numbers, the first
one is written on top of other, eg. ¼, 3/5,2/7 etc.

The top number is called the Numerator, it is the number of parts one has.
The bottom number is called the Denominator, it is the number of parts the
whole is divided into.

Numbers which can be expressed as where (a) x y, (b) x ≠ 0, y ≠0 (c) y≠

1 (d) there is no common factor. Example: , , etc.

Equivalent Fraction : A fraction can have many different appearances and


these are called Equivalent Fraction. Example: and and .

We cannot tell whether two fractions are the same until we reduce them to
their lowest terms.

Improper Fraction : An improper fraction is a fraction with the numerator


larger than or equal to the denominator. Example : , , etc.

Any natural number can be converted into Improper Fraction.

Mixed Number : A mixed number is natural number and a fraction


together. An improper fraction can be converted to a mixed number and vice
versa.

6
19. Inverse Numbers : Numbers who have negative power are called Inverse
Numbers. Example : 3-2 , 4-5 etc.
20. Even Number : Even numbers can be divided evenly into groups of two(
or any natural number which is divisible by 2 is called even number. Example
:2, 4, 8, 10,22 etc.
21. Odd Number : Odd numbers can not be divided evenly into groups of
two or any natural number which is not divisible by 2 is called an odd number.
Example : 1, 3, 5, 7, 111 etc.
22.Rational numbers : Numbers which can be represented as where x and y

are integers and y 0. Eg. 1, 2, -1, -2, , - !$etc.

Special characteristics of rational numbers:


a. All natural numbers are rational.
b. Rational number can be expressed either as a terminating decimal or a
repeating decimal.
c. Every terminating decimal is a rational number.
d. Every repeating decimal is a rational number.

23. Irrational Number : Numbers which can not be represented as ratio of two
natural numbers, are called irrational number.!Example : , 5 etc.

An irrational number is a non repeating and non terminating decimal.

! and ! are irrational numbers.

History of Irrational Numbers

Apparently Hippasus (one of Pythagoras' students) discovered irrational numbers


when trying to represent the square root of 2 as a fraction (using geometry, it is
thought). Instead he proved you couldn't write the square root of 2 as a fraction
and so it was irrational.

However Pythagoras could not accept the existence of irrational numbers, because
he believed that all numbers had perfect values. But he could not disprove
Hippasus' "irrational numbers" and so Hippasus was thrown overboard and
drowned!

7
24.Decimal form of a rational number : When we express a rational number
in decimal, then either the decimal will be exact, as = 0.5 (called terminating

decimal) or it will not, as = 0.333…. (called recurring decimal).

8
L.C.M & H.C.F
Factors and Multiples: If a number `m' divides
another number `n' exactly, then we say that HIGHEST COMMON FACTOR (H.C.F)
`m' is a factor of `n' and that `n' is a multiple
The highest common factor of two or
of `m'.
more numbers is the greatest number which
eg. 3 is a factor of 12 and therefore 12 divides each of them exactly.
is a multiple of 3.
eg. Find the H.C.F. of 24 and 56
Least Common Multiple (L.C.M.)
Factors of 24 = 1, 2, 3, 4, 6, 8, 12, 24
L.C.M. is the least non-zero number in com-
Factors of 56 = 1, 2, 4, 7, 8, 14, 28, 56
mon multiples of two or more numbers.
Common factors of 24 and 56 are 1, 2, 4, 8
Multiple of 6 = 6, 12, 18, 24, 30, ........
Multiple of 8 = 8, 16, 24, 32, 40, ........  H.C.F. of 24 and 56 = 8
Factorisation Method: H.C.F. can be found by
Common Multiple of 6 and 8 = 24, 48
resolving the given numbers into prime fac-
...............
tors and then taking the product of least pow-
Least Common Multiple = 24 ers of all common factors, that occur in these
Factorisation Method: numbers.
Find the L.C.M. of 12, 27 and 40
Eg. Find H.C.F. of 48, 108, 140
Factors of 12
Factors of 48 2 48 2 108 2 140
= 2x2x3 = 2 2 x3 2 12 3 27 2 40
= 2x2x2x2x3 2 24 2 54 2 70
Factors of 27 2 6 3 9 2 20
2 12 3 27 5 35
= 3x3x3 = 3 3
3 3 3 3 2 10 = 2 4 x3
2 6 3 9 7
Factors of 40 1 1 5 5 Factors of 108
3 3
= 2x2x2x5 = 2 3 x5 1 = 2x2x3x3x3 = 2 2 x3 3
 L. C. M .  2 3 x 3 3 x 5  1 0 8 0 Factors of 140
SHORT CUT METHOD = 2x2x5x7 = 2 2 x 5 x 7
(Division Method) H.C.F. = 2 2 = 4
Find the L.C.M. of 12, 27, 40 Division Method
Find the H.C.F. of 48, 108, 140
2 12, 27, 4 0
2 4 8 , 1 08 , 1 4 0
2 6, 27, 2 0
2 24, 54, 70
3 3, 27, 1 0
12, 27, 35
1, 9,1 0
H.C.F.=2x2= 4

L. C. M.  2x2x3x9x10 = 1080

9
QUICKER & SHORT CUT METHOD L.C.M. of 1, 5 and 10 is 10
Find the H.C.F. of 777 and 1147 H.C.F of 3, 6, 9 and 27 is 3
777) 1 1 4 7 (1 10
777 L.C.M. of given fractions =
3
370) 777(2
1 3 5 7 9
740 3 . Find the H.C.F. of , , , ,
2 4 6 8 10
37) 370 (10
Ans: H.C.F. of fractions
370
0 H. C. F. of numerat ors
= L. C. M. of deno min ators
H.C.F. of 777 and 1147 is 37
* The product of two given numbers is equal H.C.F. of 1, 3, 5, 7 and 9 is 1
to the product of their H.C.F. and L.C.M.
L.C.M of 2, 4, 6, 8 and 10 is 120
L.C.M. of two numbers
1
Product of numbers H.C.F. of given fractions =
= H. C. F. of numbers 120
4 . The L.C.M. of two number is 2310. Their
L.C.M. of given fractions H.C.F. is 30. If one number is 210, the other
is:
L. C. M. of numerators
= H. C. F. of deno min ators Ans: The other number

L. C. M. xH. C. F. 2310 x 30
H.C.F of given fractions =  330
given number = 210
H. C. F. of numerators
= L. C. M. of deno min at ors 5 . The H.C.F. and L.C.M. of two numbers
are 44 and 264 respectively. If the first num-
 The L.C.M of a given set of numbers ber is divided by 2, the quotient is 44, The
would be either the highest or higher than other number is
the highest of the given numbers. Ans: First number = 2x44 = 88
 The H.C.F. of a given set of numbers
would be either the lowest or lower than 44 x264
Second number =  132
the lowest. 88
Solved Examples 6 . The least square number which is divis-
1 . Find the L.C.M. of 125,64,8 and 3. ible by 6, 8 and 15 is:
Ans : Given numbers are 5 3, 26, 23 and 3 Ans: The least number divisible by 6, 8 and 15
is their L.C.M. which is 120
 L.C.M. 5 3x2 6x3 = 24,000
Now 120 = 2x2x2x3x5
1 5 5 10
2 . Find the L.C.M. of , , , ? To make it a perfect square, it must be
3 6 9 27 multiplied by 2x3x5
Ans: L.C.M. of fractions Required Number=120x2x3x5=3600
L. C. M. of numerat ors 7 . The least number of square tiles
= H. C. F. of deno min ators required to pave the ceiling of a room 15m
17cm long and 9m. 2cm broad is:
Ans: Size of largest square tile

10
= H.C.F. of 1517 cm and 902 cm
2 3 4
= 41 cm. 4. Find the L.C.M. of , and
5 10 15
 Least number of tiles required
1 2
Areaof the room a) b) 2
= 30 5
Areaof one tile
24 2
1517 x 902 c) d)
=  814 750 5
41 x 41
8. Find the least number which when divided 4 3 7
5. Find the H.C.F. of , and
separately by 15, 20, 36 and 48 leaves 3 as re- 5 10 15
mainder in each case.
Ans : Required number 1 84
a) b)
= L.C.M. of (15,20,36 and 48) +3 5 5
= 720 + 3 = 723
84 1
9 . Find the greatest number that will divide c) d)
197 and 269 and leaves 5 as remainder in
30 30
each case. 6. If the L.C.M of x and y is z, their H.C.F.
Required number = H.C.F. of [(197-5) is.
and (269-5)] xy
= H.C.F. of (192 and 264) = 8 a) b) xyz
z
1 2 . Five bells begin to toll together and toll
respectively at intervals of 6,7,8,9 and 12 sec- x+ y z
onds. How many times they will toll together c) d)
z xy
in one hour, excluding the one at the start?
Ans: L.C.M. of 6,7,8,9 and 12 7 . H.C.F of two numbers is 24 and their
L.C.M is 1080. If one of the numbers is 120,
= 2x2x3x7x2x3 = 504
find the other.
ie, The bells will toll together after each 504
a) 2 1 6 b) 5 3 2 c) 1 0 8 d) 8 2 0
seconds. In one hour, they will toll together
8 . L.C.M. of 2.5, 0.5 and 0.175 = ?
60 x60 a) 2 . 5 b) 0 . 5
  7 times
504 c) 0. 17 5 d) 1 7. 5
PRACTICE TEST 9 . H.C.F. of two numbers is 24 and their
1. Find the L.C.M of 12, 15, 18 and 27. L.C.M is 1344. If the difference between the
a) 1, 08 0 b) 5 4 0 numbers is 80, their sum is:
c) 2 7 0 d) 7 6 0 a) 3 6 8 b) 3 5 6
2. Find the H.C.F. of 72, 48 and 30. c) 3 3 2 c) 3 0 4
a) 3 0 b) 1 2 c) 6 d) 3 1 0 . Find the greatest number which can di-
vide 1354, 1866 and 2762 leaving the same
3. Find the L.C.M. of 2 2x33x53 and
remainder 10 in each case.
2 3 x3 2 x5.
a) 6 4 b) 1 2 4 c) 1 5 6 d) 2 6 0
a) 27 , 0 00 b) 1 8 0
1 1 . Find the least perfect square which is di-
c) 3 6 d) 13 , 5 00

11
7 1 3 5 5 7 3 1
visible by 3, 4, 5, 6 and 8. a) , , , b) , , ,
8 3 4 6 6 8 4 3
a) 2 5 0 0 b) 1 2 0 0
c) 3 6 0 0 d) 9 0 0 3 7 1 3 1 3 5 7
c) , , , d) , , ,
1 2 . The least number which when divided by 4 8 3 4 3 4 6 8
15, 27, 35 and 42 leaves in each case a re-
mainder 7 is: Qn: 18- 20 Write in descending order
a) 1 8 9 7 b) 1 9 8 7
1 2 3 1
c) 1 8 8 3 d) 2 0 0 7 18. , , ,
3 5 4 6
1 3 . Two containers contain 60 and 165 litres
of milk respectively. Find the maximum capac- 1 2 3 1 1 2 1 3
ity of a container which can measure the milk a) , , , b) , , ,
3 5 4 6 6 5 3 4
in each container an exact number of times
(in litres) 2 3 1 1 3 2 1 1
c) , , , d) , , ,
a) 1 5 b) 3 c) 5 d) 1 0 5 4 3 6 4 5 3 6
1 4 . Two baskets contain 195 and 250 ba- 5 7 11 3
nanas respectively, which are distributed in 19. , , ,
6 8 12 10
equal number among children. Find the largest
number of bananas that can be given, so that 5 7 11 3 7 5 11 3
3 bananas are left over from the first basket a) , , , b) , , ,
6 8 12 10 8 6 12 10
and 2 from the second.
a) 4 b) 1 8 c) 8 d) 6 11 7 5 3 7 5 11 3
c) , , , d) , , ,
Qn: (15- 18) :- Write in ascending order 12 8 6 10 8 6 12 10

1 2 3 3 5 11 5 7
15. , , , 20. , , ,
2 5 4 2 3 9 6 12

2 1 3 3 3 1 2 3 5 11 5 7 11 5 7 5
a) , , , b) , , , a) , , , b) , , ,
5 2 4 2 4 2 5 2 3 9 6 12 9 3 12 6

3 3 1 2 3 1 2 3 5 11 5 7 11 5 5 7
c) , , , d) , , , c) , , , d) , , ,
2 4 2 5 2 2 5 4 3 9 6 12 9 6 3 12

5 11 5 7
16. , , , Qn 21-23 Find the greatest of the given frac-
3 9 6 12
tions
11 7 5 5 7 5 11 5
a) , , , b) , , , 2 4 3 3
9 12 3 6 12 6 9 3 21. , , ,
3 15 5 4
5 7 11 5 5 11 5 7 4 3
c) , , , d) , , , a) b)
6 12 9 3 3 9 6 12 15 4
5 7 3 1 3 2
17. , , , c) d)
6 8 4 3 5 3
5 6 13 9
22. , , ,
8 11 22 13

12
5 6 and 96 cm in length. The least length of
a) b)
8 11 cloth (in metres) that can be measured
exact number of times using any of the
13 9 three rods is
c) d)
22 13 a) 0.96m b) 9.6m
c) 96 m d) 96 0m
3 5 2 8
23. , , , 29. The largest number, which exactly divides
4 7 3 11 the product of any three consecutive in-
3 5 tegers is
a) b) a) 2 b) 3 c) 6 d) 1 2
4 7 30. The L.C.M. of two numbers is 63 and their
2 8 H.C.F. is 9. If one of the numbers is 27,
c) d) the other number will be
3 11 a) 9 b) 2 1 c) 1 7 d) 1 8 9
Qn: (24 - 26) Find the smallest of the given 31. The HCF of two numbers is 32 and their
fraction. product is 10240. Find their L.C.M?
2 5 9 9 7 a) 6 4 0 b) 3 2 0 c) 3 2 4 d) 2 3 0
24. , , , , 32. A gardener had a number of shrubs to
3 7 13 14 4 plant in rows. At first he tried to plant 8,
9 2 then 12 and then 16 in a row but he had
a) b) always 3 shrubs left with him. On trying
14 3
7 he had none left. Find the total number
7 5 of shrubs.
c) d) a) 1 4 7 b) 1 5 0 c) 1 3 7 d) 1 5 4
4 7
33. Six bells commencing tolling together and
11 14 17 23 29 toll at intervals of 2,4,6,8,10 and 12 sec-
25. , , , ,
14 17 20 26 32 onds respectively. In 30 minutes, how
many times do they toll together.
29 11 a) 1 7 b) 1 5 c) 1 6 d) 2 0
a) b)
32 14 34. In a seminar the number of participants
in Hindi, English and Mathematics are 60,
17 14 84 and 108 respectively. Find the mini-
c) d)
20 17 mum number of rooms required, where
in each room the same number of par-
5 3 5 6
26. , , , ticipants are to be seated; and all of them
6 4 8 7 being in the same subject.
3 6 a) 2 0 b) 2 2 c) 2 5 d) 2 1
a) b)
4 7
ANSWERS TO PRACTICE TEST - 6
5 5
c) d) 1. (b) 2. (c) 3. (a) 4. (b) 5. (d)
8 6
2 7 . A heap of stones can be made in groups 6. (a) 7. (a) 8. (d) 9. (a) 10. (a)
of 21 but when made up into groups of 11. (c) 12. (a) 13. (a) 14. (c) 15. (a)
16, 20, 25 and 45 there are 3 stones
left in each case, The number of stones 16. (b) 17. (d) 18. (d) 19. (c) 20. (a)
in the heap is 21. (b) 22.(d) 23. (a) 24. (a) 25. (b)
a) 3 6 0 0 b) 3603
c) 7 2 0 0 d) 7203 26. (c) 27. (d) 28.(b) 29. (c) 30. (b)
2 8 . Three measuring rods are 64cm, 80cm 31. (b) 32. (a) 33. (b) 34. (d)

13
UNIT 2

Simultaneous linear
equations
The purpose of this section is to look at the solution of simultaneous linear equations. We will
see that solving a pair of simultaneous equations is equivalent to finding the location of the
point of intersection of two straight lines.
In order to master the techniques explained here it is vital that you undertake plenty of practice
exercises so that all this becomes second nature. To help you to achieve this, the unit includes
a number of such exercises.

After reading this text, and/or viewing the video tutorial on this topic, you should be able to:

• solve pairs of simultaneous linear equations

• recognise that this is equivalent to finding the point of intersection of two straight line
graphs

Contents
1. Introduction 2
2. Solving simultaneous equations - method of substitution 4
3. Solving simultaneous equations - method of elimination 6
4. Examples 6

14
1. Introduction
The purpose of this section is to look at the solution of elementary simultaneous linear equations.
Before we do that, let’s just have a look at a relatively straightforward single equation. The
equation we are going to look at is
2x − y = 3
This is a linear equation. It is a linear equation because there are no terms involving x2 , y 2 or
x × y, or indeed any higher powers of x and y. The only terms we have got are terms in x, terms
in y and some numbers. So this is a linear equation.
We can rearrange it so that we obtain y on its own on the left hand side. We can add y to each
side so that we get
2x = 3 + y
Now let’s take 3 away from each side.
2x − 3 = y
This gives us an expression for y: namely y = 2x − 3.
Suppose we choose a value for x, say x = 1, then y will be equal to:
y = 2 × 1 − 3 = −1
Suppose we choose a different value for x, say x = 2.
y =2×2−3=1
Supppose we choose another value for x, say x = 0.
y = 2 × 0 − 3 = −3
For every value of x we can generate a value of y.
We can plot these as points on a graph. We can plot the first as the point (1, −1). We can plot
the second one as the point (2, 1), and the third one as the point (0, −3) and so on. Plotting
the points on a graph, as shown in Figure 1, we see that these three points lie on a straight line.
This is the line with equation y = 2x − 3. It is a straight line and this is another reason for
calling the equation a linear equation.

2
y = 2x − 3
1

0 1 2 x

-1

-2
-3

Figure 1. Graph of y = 2x − 3.

15
Suppose we take a second linear equation 3x + 2y = 8 and plot its graph on the same figure. A
quick way to achieve this is as follows.
When x = 0, 2y = 8, so y = 4. Therefore the point (0, 4) lies on the line.

8 2
When y = 0, 3x = 8, so x = = 2 . Therefore the point (2 23 , 0) lies on the line.
3 3

Because this is a linear equation we know its graph is a straight line, so we can obtain this by
joining up the points. Both straightline graphs are shown in Figure 2.

y
4

3 3x + 2y = 8

2
y = 2x − 3
1

0 1 2 x

-1

-2
-3

Figure 2. Graphs of y = 2x − 3 and 3x + 2y = 8

When we solve a pair of simultaneous equations what we are actually looking for is the intersec-
tion of two straight lines because it is this point that satisfies both equations at the same time.
From Figure 2 we see that this occurs at the point where x = 2 and y = 1.
Of course it could happen that we have two parallel lines; they would never meet, and hence
the simultaneous equations would not have a solution. We shall observe this behaviour in one
of the examples which follows.

Key Point
When solving a pair of simultaneous linear equations we are, in fact, finding a common point
- the point of intersection of the two lines.

16
2. Solving simultaneous equations - method of substitution
How can we handle the two equations algebraically so that we do not have to draw graphs? We
are going to look at two methods of solution. In this Section we will look at the first method -
the method of substitution.
Let us return to the two equations we met in Section 1.

2x − y = 3 (1)
3x + 2y = 8 (2)

By rearranging Equation (1) we find

y = 2x − 3 (3)

We can now substitute this expression for y into Equation (2).

3x + 2(2x − 3) = 8
3x + 4x − 6 = 8
7x − 6 = 8
7x = 14
x = 2

Finally, using Equation (3), y = 2 × 2 − 3 = 1. So x = 2, y = 1 is the solution to the pair of


simultaneous equations.
This solution should always be checked by substituting back into both original equations to
ensure that the left- and right- hand sides are equal for these values of x and y. So, with x = 2,
y = 1, the left-hand side of Equation (1) is 2(2) − 1 = 3, which is the same as the right-hand
side. With x = 2, y = 1, the left-hand side of Equation (2) is 3(2) + 2(1) = 8, which is the same
as the right-hand side.

Example

Let’s have a look at another example using this particular method.

The example we are going to use is

7x + 2y = 47 (1)
5x − 4y = 1 (2)

Now we need to make a choice. We need to choose one of these two equations and re-arrange it
to obtain an expression for y, or if we wish, for x

The choice is entirely ours and we have to make the choice based upon what we feel will be
the simplest. Looking at a pair of equations like this, it is often difficult to know which is the
simplest.

17
Let’s choose Equation (2) and rearrange it to find an expression for x.

5x − 4y = 1
5x = 1 + 4y by adding 4y to each side
1 + 4y
x = by dividing both sides by 5
5

We now use this expression for x and substitute it in Equation (1).

 
1 + 4y
7 + 2y = 47
5

Now multiply throughout by 5. Why? Because we want to get rid of the fraction and the way
to do that is to multiply everything by 5.

7(1 + 4y) + 10y = 235

Now we need to multiply out the brackets

7 + 28y + 10y = 235

Gather the y’s and subtract 7 from each side to get

38y = 228

So
228
y= =6
38
So we have established that y = 6. Having done this we can substitute it back into the equation
that we first had for x.
1 + 4y 1 + 24
x= =
5 5
and so
x=5
So again, we have our pair of values - our solution to the pair of simultaneous equations. In order
to check that our solution is correct these values should be substituted into both equations to
ensure they balance. So, with x = 5, y = 6, the left-hand side of Equation (1) is 7(5)+2(6) = 47,
which is the same as the right-hand side. With x = 5, y = 6, the left-hand side of Equation (2)
is 5(5) − 4(6) = 1, which is the same as the right-hand side.
Exercises
1. Solve the following pairs of simultaneous equations:
y = 2x + 3 y = 3x − 1 6x + y = 4
a) b) c)
y = 5x − 3 2x + 4y = 10 5x + 2y = 1

x − 3y = 1 2x + 13 y = 1 4x + 3y = 5
d) e) f)
2x + 5y = 35 3x + 5y = 6 2x − 34 y = 1

18
3. Solving simultaneous equations - method of elimination
We illustrate the second method by solving the simultaneous linear equations:
7x + 2y = 47 (1)
5x − 4y = 1 (2)
We are going to multiply Equation (1) by 2 because this will make the magnitude of the coeffi-
cients of y the same in both equations. Equation (1) becomes
14x + 4y = 94 (3)
If we now add Equation (2) and Equation (3) we will find that the terms involving y disappear:

5x − 4y = 1
+
14x + 4y = 94
19x = 95
and so
95
x= = 5
19
Now that we have a value for x we can substitute this into Equation (2) in order to find y.
Substituting
5x − 4y = 1
5 × 5 − 4y = 1
25 = 4y + 1
24 = 4y
y = 6

The solution is x = 5, y = 6.

4. Examples
Solve the simultaneous equations
3x + 7y = 27 (1)
5x + 2y = 16 (2)
We will multiply Equation (1) by 5 and Equation (2) by 3 because this will make the coefficients
of x in both equations the same.
15x + 35y = 135 (3)
15x + 6y = 48 (4)
If we now subtract Equation (4) from Equation (3) we can eliminate the terms involving x.

19
15x + 35y = 135

15x + 6y = 48
29y = 87
from which
87
y= = 3
29
If we substitute this result in Equation (1) we can find x.

3x + 7y = 27
3x + 21 = 27
3x = 6
x = 2

As before, the solution should be checked by substitution into the original equations. So, with
x = 2, y = 3, the left-hand side of Equation (1) is 3(2) + 7(3) = 27, which is the same as the
right-hand side. With x = 2, y = 3, the left-hand side of Equation (2) is 5(2) + 2(3) = 16, which
is the same as the right-hand side.
All the examples that we have looked at so far have all had whole number coefficients; let’s have
a look at a couple that don’t look like the ones we have just done.

Example
Solve the simultaneous equations

x = 3y
x
− y = 34
3
First of all let us rearrange the first equation so that x and y terms are on the left. We will also
multiply the second equation by 3 to remove the fraction. These operations give

x − 3y = 0
x − 3y = 102

Notice that the terms on the left in both equations are exactly the same. If we subtract the
equations we will find 0 = −102. This does not make sense. Remember right at the beginning
of this unit we explained that if two lines are parallel they will not intersect. This is the case
here. There are no solutions.

Example

x y
− = 0 (1)
5 4
1
3x + y = 17 (2)
2
Observe that if both sides of Equation (1) are multiplied by 20 we can remove the fractions:

4x − 5y = 0 (3)

20
If Equation (2) is multiplied by 2 we can remove the fraction there too.
6x + y = 34 (4)
Now multiply Equation (4) by 5:
30x + 5y = 170 (5)
We can now add (3) and (5) to obtain
34x = 170
170
x= =5
34
Substituting this value into Equation (1) gives
x y
− = 0
5 4
y
1− = 0
4
from which y = 4.
So the solution is: x = 5, y = 4. As before, this should be checked by substitution into the
original equations. So, with x = 5, y = 4, the left-hand side of Equation (1) is 55 − 44 = 0, which
is the same as the right-hand side. With x = 5, y = 4, the left-hand side of Equation (2) is
3(5) + 12 (4) = 17, which is the same as the right-hand side.
To summarise:
A pair of simultaneous equations represent two straight lines. In effect when we solve them we
are looking for the point where the two straight lines intersect. The method of elimination is
much better to use than the first method.
Remember the answer you get can always be checked by substituting the pair of values into the
original equations.
Exercises
2. Use elimination to solve the following pairs of simultaneous equations.
5x + 3y = 9 2x − 3y = 9 x + 7y = 10
a) b) c)
2x − 3y = 12 2x + y = 13 3x − 2y = 7

5x + y = 10
1
3
x+ y = 10
3
3x − 2y = 52
d) e) f)
7x − 3y = 14 2x + 1
4
y = 11
4
1
3
x + 3y = − 43
3. Solve the following pairs of simultaneous equations by a method of your choice.
x = 3y x = 13 y 7x + 3y = −15
a) b) c)
4x − 5y = 35 2y − 6x = 9 12y − 5x = 39
Answers
a) x = 2, y = 7 b) x = 1, y = 2 c) x = 1, y = −2
1.
d) x = 10, y = 3 e) x = 1/3, y = 1 f) x = 3/4, y = 2/3
a) x = 3, y = −2 b) x = 6, y = 1 c) x = 3, y = 1
2.
d) x = 2, y = 0 e) x = 1, y = 3 f) x = 1/2, y = −1/2
3. a) x = 15, y = 5 b) no solution c) x = −3, y = 2

21
 1 
(vii).   , 3 (viii). {-a, -b} (ix).
 2 
(x). (xi). (xii).

16 1  113
Q.2. (i). {2, 4} (ii). {2,  } (iii). { }
3 2
1
(iv). {– a, – b} (v). {3, } (vi). {-1, 25}
3

b
(vii). {3b,  } (viii) { (a + b), (a – b)}
2
3 1  53 11  13
Q.3. (i). { , 3 } (ii). { } (iii). { }
2 2 6
25 1  1
(iv). { ,  } (v). { m  n,  2(m  n) } (vi) 1, 
2 2  m
2 3  6  3 6  3 
(vii). {  , } (viii). {  b, a } (ix)  , 
a b  3 3 

Q.4. 7,-8 Q.5. 8.465 seconds Q.6. 10.6 m

1.8 Classification of Numbers


1. The Set N of Natural Numbers:
Whose elements are the counting, or natural numbers:
N = {1, 2, 3, - - - - - - - - }

2. The Set Z of Integers:


Whose elements are the positive and negative whole numbers and zero:
Z = {- - - - - - , -2, -1, 0, 1, 2, - - - - - - }

3. Whose elements are all those numbers that


a
can be represented as the quotient of two integers , where b ≠ 0. Among the
b
3 18 5 9
elements of Q are such numbers as  , , ,  . In symbol
4 27 1 1
a 
Q =  | a, b  Z, b  0 
b 
Equivalently, rational numbers are numbers with terminating or repeating
decimal representation, such as
1.125, 1.52222, 1.56666, 0.3333

22
4.
Whose elements are the numbers with decimal representations that are non-
terminating and non-repeating. Among the elements of this set are such numbers as
2,  7,  .
a
An irrational number cannot be represented in the form , where a, b  Z . In
b
symbols,
Q = {irrational numbers}
5. The Set R of Real Numbers:
Which is the set of all rational and irrational numbers:
R = {x | x  Q Q'}
6. The set I of Imaginary Numbers:
Whose numbers can be represented in the form x + yi, where x and y are real
numbers,  √
I = {x + yi | x, y  R, y  0, i = -1}
If x = 0, then the imaginary number is called a pure imaginary number.
An imaginary number is defined as, a number whose square is a negative i.e,
1, -3, -5
7. The set C of Complex Numbers:
Whose members can be represented in the form x + y i, where x and y real
numbers and i = -1 :
C = {x + yi | x, y  R, i = -1}
With this familiar identification, the foregoing sets of numbers are related as
indicated in Fig. 1.
Natural numbers

Imaginary numbers Integers Zero

Negative of
Complex numbers Rational numbers natural numbers

Real numbers Non-integers

Irrational numbers
Fig. 1
Hence, it is clear that N  Z  Q  R  C

1.9 Nature of the roots of the Equation ax2 + bx + c = 0


The two roots of the Quadratic equation ax2 + bx + c = 0 are:

23
b± b 2  4ac
x=
2a
The expression b2 – 4ac which appear under radical sign is called the
Discriminant (Disc.) of the quadratic equation. i.e., Disc = b2 – 4ac
The expression b2 – 4ac discriminates the nature of the roots, whether they are
real, rational, irrational or imaginary. There are three possibilities.
(i)b2 – 4ac < 0 (ii) b2 – 4ac = 0 (iii) b2 – 4ac > 0

(i) If b2 – 4ac < 0, then roots will be imaginary and unequal.


(ii) If b2 – 4ac = 0, then roots will be real, equal and rational.
(This means the left hand side of the equation is a perfect square).
(iii) If b2 – 4ac > 0, then two cases arises:
(a) b2 – 4ac is a perfect square, the roots are real, rational and unequal.
(This mean the equation can be solved by the factorization).
(b) b2 – 4ac is not a perfect square, then roots are real, irrational and
unequal.
Example 1:
Find the nature of the roots of the given equation
9x2 + 6x + 1 = 0
Solution:
9x2 + 6x + 1 = 0
Here a = 9, b = 6, c = 1
Therefore , Discriminant = b2 – 4ac
= (6)2 – 4(9) (1)
= 36 – 36
=0
2
Because b – 4ac = 0
 roots are equal, real and rational.
Example 2:
Find the nature of the roots of the Equation
3x2 – 13x + 9 = 0
Solution:
3x2 – 13x + 9 = 0
Here a = 3, b = -13, c = 9
Discriminant = b2 – 4ac
= (-13)2 -4(3) (9)
= 169 – 108 = 61
2
Disc = b – 4ac = 61 which is positive
Hence the roots are real, unequal and irrational.
Example 3:
For what value of “K” the roots of Kx2 + 4x + (K – 3) = 0
are equal.
Solution:
Kx2 + 4x + (K – 3) = 0
Here a = K, b = 4, c = K – 3
Disc = b2 – 4ac

24
= (4)2 – 4(K)(K – 3)
= 16 – 4K2 + 12K
The roots are equal if b2 – 4ac = 0
i.e. 16 – 4K2 + 12K = 0
4K2 – 3K – 4 = 0
2
K – 4K + K – 4 = 0
K(K – 4) + 1(K – 4) = 0
Or K = 4, –1
Hence roots will be equal if K = 4, –1
Example 4:
Show that the roots of the equation
2(a + b)x2 – 2(a + b + c)x + c = 0 are real
Solution: 2(a + b)x2 – 2(a + b + c)x + c = 0
Here, a = 2(a + b) , b = –2 (a + b + c) , c = c
2
Discriminant = b – 4ac
= [–2(a + b + c)]2 – 4[2(a + b) c]
= 4(a 2 + b2 + c2 + 2ab + 2bc + 2ac)  8(ac + bc)
= 4(a 2 + b2 + c2 + 2ab + 2bc + 2ac  2ac  2bc )
= 4(a 2 + b2 + c2 + 2ab)
= 4[(a 2 + b2 + 2ab)+ c2 ]
= 4[(a+ b)2 + c2 ]
Since each term is positive, hence
Disc > 0 Hence , the roots are real.
Example 5:
For what value of K the roots of equation 2x2 + 5x + k = 0 will be rational.
Solution:
2x2 + 5x + k = 0
Here, a = 2, b = 5, c = k
The roots of the equation are rational if
Disc = b2 – 4ac = 0
So, 52 – 4(2)k = 0
25 – 8k = 0
25
k= Ans
8
Exercise 1.2
Q1. Find the nature of the roots of the following equations
(i) 2x2 + 3x + 1 = 0 (ii) 6x2 = 7x +5
(iii) 3x2 + 7x – 2 = 0 (iv) √ √

Q2. For what value of K the roots of the given equations are equal.
(i) x2 + 3(K + 1)x + 4K + 5 = 0 (ii) x2 + 2(K – 2)x – 8k = 0
2
(iii) (3K + 6)x + 6x + K = 0 (iv) (K + 2)x2 – 2Kx + K – 1 = 0

25
Q3. Show that the roots of the equations
(i) a2(mx + c)2 + b2x2 = a2 b2 will be equal if c2 = b2 + a2m2
a
(ii) (mx + c)2 = 4ax will be equal if c=
m
(iii) x2 + (mx + c)2 = a2 has equal roots if c2 = a2 (1 + m2).

Q4. If the roots of (c2 – ab)x2 – 2(a2 – bc)x + (b2 – ac) = 0 are equal then prove that
a3 + b3 + c3 = 3abc
Q5. Show that the roots of the following equations are real
1
(i) x2 – 2 ( m+ )x + 3 = 0
m
(ii) x2 – 2ax + a2 = b2 + c2
(iii)(b2 – 4ac)x2 + 4(a + c)x – 4 = 0
Q6. Show that the roots of the following equations are rational
(i) a(b – c)x2 + b(c – a)x + c(a – b) = 0
(ii) (a + 2b)x2 + 2(a + b + c)x + (a + 2c) = 0
(iii) (a + b)x2 – ax – b) = 0
(iv) p x2 - (p – q) x – q = 0

Q7. For what value of „K‟ the equation (4–k) x2 + 2(k+2) x + 8k + 1 = 0 will be a
perfect square.
(Hint : The equation will be perfect square if Disc. b2 – 4ac = 0 )

Answers 1.2
Q1. (i)Real, rational, unequal (ii) unequal, real and rational
(iii) ir-rational, unequal, real (iv) Real, unequal, ir-rational
-11
Q2. (i) 1, (ii) -2 (iii) 1, -3 (iv) 2
9

Q7. 0, 3

1.10 Sum and Product of the Roots


(Relation between the roots and Co-efficient of ax2 + b x + c = 0)
The roots of the equation ax2 + bx + c = 0 are
b± b 2  4ac
 =
2a
b  b2  4ac
=
2a

26
Sum of roots:
Add the two roots
b+ b 2  4ac b  b 2  4ac
  = 
2a 2a
b+ b2  4ac  b  b 2  4ac
=
2a
b  b
=
2a
2b b
= = 
2a a
 Co-efficient of x
Hence, sum of roots =    =
Co-efficient of x 2
Product of roots:
 b+ b 2  4ac   b  b 2  4ac 
 =  x  =
 2a   2a 
   

b2  b2  4ac
=
4a 2
4ac
=
4a 2
c
a =
a
 Constant term
i.e. product of roots =   =
Co-efficient of x 2
Example 1:
Find the sum and the Product of the roots in the Equation 2x2 + 4 = 7x
Solution:
2x2 + 4 = 7x
2x2 – 7x + 4 = 0
Here a = 2, b = -7, c = 4
b  7 7
Sum of the roots =  =    =
a  2 2
c 4
Product of roots = = =2
a 2
Example 2:
Find the value of “K” if sum of roots of

27
5
(2k – 1)x2 + (4K – 1)x + (K + 3) = 0 is
2
Solution:
(2k – 1)x2 + (4K – 1)x + (K + 3) = 0
Here a = (2k – 1), b = 4K – 1, c = K + 3
b
Sum of roots = 
a
5 (4K - 1) 5
=  Sum of roots =
2 (2K - 1) 2
5 (2K – 1) = – 2 (4K – 1)
10K – 5 = –8K + 2
10K + 8K = 5 + 5
18K = 7
7
K=
18
Example 3:
If one root of 4x2 – 3x + K = 0 is 3 times the other, find the value of “K”.
Solution:
Given Equation is 4x2 – 3x + K = 0
Let one root be , then other will be 3 .
a
Sum of roots = 
b
(  3)
 + 3 = 
4
3
4 =
4
3
 =
16
Product of roots =
K
(3) =
4
K
32 =
4
K = 122
3
Putting the value of  = we have
16
2
 3
K = 12  
 16 
12x9 27
= =
256 64

28
Exercise 1.3
Q1. Without solving, find the sum and the product of the roots of the following
equations.
(i) x2 – x + 1 = 0 (ii) 2y2 + 5y – 1 = 0
(iii) x2 – 9 = 0 (iv) 2x2 + 4 = 7x
2
(v) 5x + x – 7 = 0
Q2. Find the value of k, given that
(i) The product of the roots of the equation
7
(k + 1)x2 + (4k + 3)x + (k – 1) = 0 is
2
(ii) The sum of the roots of the equation 3x2 + k x + 5 = 0 will be equal to
the product of its roots.
(iii) The sum of the roots of the equation 4 x2 + k x - 7 = 0 is 3.

Q3. (i)If the difference of the roots of x2 – 7x + k – 4 = 0 is 5, find the value of k and
the roots.
5
(ii) If the difference of the roots of 6x2 – 23x + c = 0 is , find the value of k
6
and the roots.
Q4. If  , β are the roots of ax2 + bx + c = 0 find the value of
1 1 
(i) 3 + β3 (ii)  2 (iii) √ √

 2

 
(iv) (v)
 
(v)
Q5. If p, q are the roots of 2x2 – 6x + 3 = 0 find the value of
(p3 + q3) – 3pq (p2 + q2) –3pq (p + q)
Q6. The roots of the equation px2 + qx + q = 0 are  and β,

Prove that √ √ √

Q7. Find the condition that one root of the equation px2 + qx + r = 0 is
square of the other.
Q8. Find the value of k given that if one root of 9x2 – 15x + k = 0 exceeds the other
by 3. Also find the roots.
Q9. If  , β are the roots of the equation px2 + qx + r = 0 then find the values of

(i) 2 + β2 (ii) (  β)2 (iii) 3β + β3

Answers 1.3
5 1 7 1 7
Q1.(i) 1, 1 (ii)  ,  (iii) 0, - 9 (iv) ,2 (v)  , 
2 2 2 5 5

29
7 9
Q2.(i) (ii)  (iii) - 12
18 5
7 3
Q3.(i) K =10, roots = 6, 1 (ii)  ,   ; c = 21
3 2
b3  3abc b2  2ac b 3abc  b3 b b2  4ac
Q4. (i) (ii) (iii)  (iv) (v)
a3 c2 ac a 2c ac
2 7
Q5. - 27 Q7. Pr (p + r)+q3 = 3pqr Q8. K = - 14, roots are  ,
3 3

q 2  2pr q 2  4pr
Q9. (i) (ii) (iii)
p2 p2

1.11 Formation of Quadratic Equation from the given roots :


Let  ,  be the roots of the Equation ax2 + bx + c = 0
b
The sum of roots =      ……… (I)
a
c
Product of roots =  .  ……… (II)
a
The equation is ax2 + bx + c = 0
b c
Divide this equation by a  x2  x+  0
a a
 b c
Or x 2     x+  0
 a a
From I and II this equation becomes

x2 - (  + β ) x +  β = 0

Or x2 – (Sum of roots) x + Product of roots = 0

Or x2 – (S) x + (P) = 0

is the required equation, where S =    and P =  


Alternate method:-
Let  ,  be the roots of the equation a x2 + b x + c = 0
i.e., x =  and x = β
 x-  = 0 and x - β = 0
 (x - ) ( x - β ) = 0

x2 -  x - β x +  β = 0

30
x2 - (  + β ) x +  β = 0

Or x2 – (Sum of roots) x + Product of roots = 0

Or x2 – S x + P = 0
is the required equation, where S =    and P =  
Example 4:
Form a quadratic Equation whose roots are 3 5, 3 5
Solution:
Roots of the required Equation are 3 5 and 3 5
Therefore S = Sum of roots = 3 5  3 5
S=0
P = Product of roots = (3 5)(3 5) = – 9 (5)
P = – 45
Required equation is
x2 – (Sum of roots) x + (Product of roots) = 0
Or x2 – Sx + P = 0
x2 – 0(x) + (–45) = 0
x2 – 0 – 45 = 0
x2 – 45 = 0
Example 5:
If  ,  are the roots of the equation ax2 + bx + c = 0, find the equation whose
 
roots are , .
 
Solution:
Because  ,  are the roots of the Equation ax2 + bx + c = 0
b
The sum of roots =     
a
b
Product of roots =   
a
 
Roots of the required equation are ,
 
Therefore ,
 
S = sum of roots of required equation = +
 
2  2
= (   )2   2   2  2

2
 b c
(   )2  2    2
= 
a a
=
 

31
b 2 2c

a 2
a b 2  2ac a
= = x
c a2 c
a
b  2ac
2
S=
ac
  
P = Product of roots of required equation = . =
  
P =1
Required equation is: x2 – Sx + P = 0
 b 2  2ac 
x  
2
 x + 1 = 0
 ac 
acx2 – (b2 – 2ac)x + ac = 0

Exercise 1.4
Q1. Form quadratic equations with the following given numbers as its roots.
(i) 2, - 3 (ii) 3 +i , 3 – i (iii) 2+ 3, 2- 3

(iv) 3+ 5,  3  5 (v) 4 + 5 i , 4  5i


Q2. Find the quadratic equation with roots
(i)Equal numerically but opposite in sign to those of the roots of the
equation 3x2 + 5x – 7 = 0
(ii)Twice the roots of the equation 5x2 + 3x + 2 = 0
(iii)Exceeding by „2‟ than those of the roots of 4x2 + 5x + 6 = 0

Q3. Form the quadratic equation whose roots are less by „1‟ than those of
3x2 – 4x – 1 = 0

Q4. Form the quadratic equation whose roots are the square of the roots of the
equation 2x2 – 3x – 5 = 0

Q5. Find the equation whose roots are reciprocal of the roots of the equation
px2 – qx + r = 0

Q6. If ,  are the roots of the equation x2 – 4x + 2 = 0 find the equation whose
roots are
1 1
(i) α2 , β2 (ii) α3 , β3 (iii)  , 
 
(iv) α + 2 ,β + 2

32
Q7. If α , β are the roots of ax2 + bx + c = 0 form an equation whose roots are
   
(i) , (ii) (iii) ,
   

Answers 1.4
Q1. (i) x2 + x – 6 = 0 (ii) x2 – 6x + 10 = 0
(iii) x2 – 4x + 1 = 0 (iv) x2 + 6x + 4 = 0 (v) x2 - 5 x + 41 = 0
Q2. (i) 3x2 – 5x – 7 = 0 (ii) 5x2 - 6x + 8 = 0
(iii) 4x2 – 11x + 12 = 0
2
Q3. 3x + 2x – 2 = 0 Q4 . 4x2 – 29x + 25 = 0
Q5. rx2 – qx + p = 0 Q6. (i) x2 – 12x + 4 = 0 (ii) x2 – 40x + 8
=0
(iii) 2x2 – 12x + 17 = 0 (iv) x2 – 8x + 14 = 0
Q7. (i) acx – (b – 2ac)x + ac = 0 (ii) a cx + (b3 – 3abc)x + ac2 = 0
2 2 2 2

(iii) cx2 – (2c – b)x + (a – b + c) = 0

Summary
Quadratic Equation:
An equation of the form ax2 + bx + c = 0, a ≠ 0, where a, b , c  R and x is a
variable, is called a quadratic equation.
If  ,  are its roots then
b+ b 2  4ac b  b 2  4ac
 ,
2a 2a
Nature of Roots:
(i) If b2 – 4ac > 0 the roots are real and distinct.
(ii) If b2 – 4ac = 0 the roots are real and equal.
(iii) If b2 – 4ac < 0 the roots are imaginary.
(iv) If b2 – 4ac is a perfect square, roots will be rational, otherwise irrational.
Relation between Roots and Co-efficients
If  and  be the roots of the equation ax2 + bx + c = 0
-b
Then sum of roots =    =
a
c
Product of roots =  =
a
Formation of Equation
If  and  be the roots of the equation ax2 + bx + c = 0 then we have
x2 – (sum of roots)x + (product of roots) = 0

33
Short Questions

Write Short answers of the following questions:

Solve the following quadratic equations by factorization

Q.1 x2 + 7x + 12 = 0

Q2. x2 – x = 2

Q3. x(x + 7) = (2x – 1) ( x + 4)

Q4. 6x2 – 5x = 4

Q5. 3x2 + 5x = 2

Q6. 2x2 + x = 1

Q7. m x2 + (1 + m ) x + 1 =0

Solve the following equations by completing the square:

Q8. x2 – 2 x – 899 = 0

Q9. 2x2 +12x – 110 =0

Q10. x2 + 5x – 6 = 0

Q11. x2 – 6x + 8 = 0

Solve the following equations by quadratic formula :


Q12. 4x2 +7x – 1 = 0

Q13. 9 x2 – x – 8 = 0

Q14. X2 – 3x – 18 = 0

Q15. X2 – 3x = 2x – 6

Q16. 3x2 – 5x – 2 = 0

Q17. 16 x2 + 8 x + 1 = 0

Q18 Define discriminant


Discuss the nature of the roots of the equation:

34
Q19 2 x2 – 7x + 3 =0

Q20. x2 – 5x – 2 =0

Q21. x2 + x + 1 = 0

Q22. x2 – 2 2 x + 2 = 0

Q23. 9x2 + 6x + 1 = 0

Q24. 3x2 – 13x + 9 = 0

For what value of K the roots of the following equations are equal:
Q25 Kx2 + 4x + 3 = 0

Q26. 2x2 + 5x + K = 0
Q27 Prove that the roots of the equation

(a + b) x2 – a x - b = 0 are rational

Q28 Write relation between the roots and the coefficients of the quadratic equation

a x2 + b x + c = 0

Q.29 If the sum of the roots of 4x2 + k x – 7 =0 is 3, Find the value of k.

Q.30 Find the value of K if the sum of the roots of equation

(2k – 1)x2 + (4k – 1) x + (K + 3) =0 is 5/2

Find the sum and product of the roots of following equations:

Q31 7x2 -5x + 4 = 0

Q32. x2 – 9 = 0

Q33. 9x2 + 6x + 1 = 0

Q34. For what value of k the sum of roots of equation 3x2 + kx + 5 = 0

may be equal to the product of roots?

Q35. If α , β are the roots of x2 – px – p – c = 0 then prove that (1+ α) (1+ β) = 1 – c

Write the quadratic equation for the following equations whose roots are :

Q.36 -2, -3

Q37. ἰ 3, -ἰ 3

35
Q38. -2+ 3, -2– 3

Q.39 Form the quadratic equation whose roots are equal numerically but opposite in

sign to those of 3x2–7x–6 = 0

If α, β are the roots of the equation x2 – 4 x + 2 = 0 find equation whose roots are:

1 1
Q40. α, β

Q41 -  , -β

Answers
Q1. {- 3 , - 4} Q2 {- 1 , 2} Q3 { 2 , -2} Q4 {4/3 , - ½} Q5 {1 , -6}

Q6 {-1 , ½} Q7 { -1 , - 1/m} Q8 {-29 , 31} Q9 {-11 , 5} Q10{1 , – 6}

√ √
Q11 {2 , 4} Q12. {1 , -6} Q13 { Q14 {-8/9 , 1}

Q15 {6 , -3} Q16 {2 , 3} Q17 {2 , -1/3} Q18 {-1/4 }

Q19. Roots are rational , real and unequal

Q20 Roots are irrational , real and unequal

Q21 Roots are imaginary Q22 Roots are equal and real

Q23 Roots are equal and real Q24 Roots are unequal , real and irrational

Q25. K = 4/3 Q26. K = 5 Q29 K = -12 Q30. K = 7/18

Q31 S = 5/7 , P = 4/7 Q32 S=0,P=-9 Q33 S = -2/3 , 1/9


2 2
Q34 K = - 5 Q36 x +5 x + 6 =0 Q37 x +3=0 Q38 x2 + 4x + 1 = 0

Q39 3x2 + 7 x – 2 = 0 Q40 2x2 – 4 x + 1 = 0 Q41 x2 + 4 x + 2 = 0

36
Objective Type Questions
Q1. Each question has four possible answers .Choose the correct
answer and encircle it .
__1. The standard form of a quadratic equation is:
(a) ax2 + bx = 0 (b) ax2 = 0
(c) ax2 + bx + c = 0 (d) ax2 + c = 0
2
__2. The roots of the equation x + 4x – 21 = 0 are:
(a) (7, 3) (b) (–7, 3)
(c) (–7, –3) (d) (7, –3)
__3. To make x2 – 5x a complete square we should add:
25 25 25
(a) 25 (b) (c) (d)
4 9 16
__4. The factors of x2 – 7x + 12 = 0 are:
(a) (x – 4)(x + 3) (b) (x – 4)(x – 3)
(c) (x + 4)(x + 3) (d) (x + 4)(x – 3)

__5. The quadratic formula is:


b  b 2  4ac b  b2 +4ac
(a) (b)
2a 2a
b  b 2  4ac b  b2 +4ac
(c) (d)
2a 2a
__6. A second degree equation is known as:
(a) Linear (b) Quadratic
(c) Cubic (e) None of these
__7. Factors of x3 – 1 are:
(a) (x – 1)(x2 – x – 1) (b) (x – 1)(x2 + x + 1)
2
(c) (x – 1)(x + x – 1) (d) (x – 1)(x2 – x + 1)
__8. To make 49x2 + 5x a complete square we must add:
2 2
 5  14 
(a)   (b)  
 14   5
2 2
5 7
(c)   (d)  
7 5
__9. lx2 + mx + n = 0 will be a pure quadratic equation if:
(a) l=0 (b) m=0
(c) n=0 (d) Both l, m = 0
2
__10. If the discrimnant b – 4ac is negative, the roots are:
(a) Real (b) Rational
(c) Irrational (d) Imaginary
__11. If the discriminant b2 – 4ac is a perfect square, its roots will be:
(a) Imaginary (b) Rational
(c) Equal (d) Irrational
__12. The product of roots of 2x2 – 3x – 5 = 0 is:

37
5 5
(a)  (b)
2 2
2 2
(c) (d) 
5 5
__13. The sum of roots of 2x2 – 3x – 5 = 0 is:
3 3
(a)  (b)
2 2
2 2
(c) (d) 
3 3
__14. If 2 and – 5 are the roots of the equation, then the equations is:
(a) x2 + 3x + 10 = 0 (b) x2 – 3x – 10 = 0
(c) x2 + 3x – 10 = 0 (d) 2x2 – 5x + 1 = 0
__15. If ± 3 are the roots of the equation, then the equation is:
(a) x2 – 3 = 0 (b) x2 – 9 = 0
2
(c) x +3=0 (d) x2 + 9 = 0
__16. If „S‟ is the sum and „P‟ is the product of roots, then equation is:
(a) x2 + Sx + P = 0 (b) x2 + Sx – P = 0
(c) x2 – Sx + P = 0 (d) x2 – Sx – P = 0
2
__17. Roots of the equation x + x – 1 = 0 are:
(a) Equal (b) Irrational
(c) Imaginary (d) Rational
__18. If the discriminant of an equation is zero, then the roots will be:
(a) Imaginary (b) Real
(c) Equal (d) Irrational
__19. Sum of the roots of ax2 – bx + c = 0 is:
c c
(a)  (b)
a a
b b
(c)  (d)
a a
__20. Product of roots of ax2 + bx – c = 0 is:
c c a a
(a) (b)  (c) (d) 
a a b b
Answers
1. c 2. b 3. b 4. b 5. c
6. b 7. b 8. a 9. b 10. d
11. b 12. a 13. b 14. c 15. b
16. c 17. b 18. c 19. d 20. b

38
 
 
FACTORING QUADRATIC EQUATIONS 

Summary
1.  Difference of squares .............................................................................................................. 1 
2.  Mise en évidence simple ......................................................................................................... 2 
3.  compounded factorization ...................................................................................................... 3 
4.  Exercises .................................................................................................................................. 7 
 

The  goal  of  this  section  is  to  summarize  the  methods  allowing  us  to  factor  quadratic 
equations,  i.e.  of  form    .  We  will  avoid  using  the  famous  discriminant 
formula  

√ 4
 
2

as  much  as  possible.  Three  methods  allow  us  to  carry  out  the  factoring  of  most 
quadratic functions.  

1. Difference of squares

There is a formula that allows for rapid factorization. When a function presents in the 
form  , it can be factored by the difference of squares formula, i.e.  

Hence,  you  need  to  obtain  the  square  root  of  the  first  and  second  term  and  multiply 
their sum with their difference. The negative sign separating the terms  and    is of 
capital  importance.  For  example,  the  rule  presented  above  cannot  be  applied  to 
, as the name difference in squares implies! 

Example

9 3 3 3

100 10 10 10

81 9 9 9

39
In the preceding examples, we chose to use values that are perfect squares. This criteria 
is  not  generally  necessary.  For  example,  although  8  is  not  a  perfect  square,  its  root  is 
well defined (it exists) and is equal to √8. 

Example 

8 √8 √8 √8  

3 √3 √3 √3

45 √45 √45 √45  

Also, it is possible that the first term is not   but 9 . Once again, you do not need to 


worry. We must find the square root of 9  instead of the square root of  . In all other 
aspects, the method remains the same. 

Example 

9 25 3 5 3 5 3 5  

16 49 4 7 4 7 4 7  

8 9 √8 3 √8 3 √8 3  

2. Simple factorization

We must avoid confusing the form of the difference of squares x2 ‐ k2  with ax2 ‐ bx. The 
presence of an   in the second term will allow us to proceed with a simple factorization. 
We can bring out the common factor, i.e. the x.  

Example 

6 6

We  factored  (placed  in  front  of  a  parenthesis)  an  since  each  of  the  terms  6  
has  at  least  an  .  Note  that  the  product  6 is  equal  to  6 .  You  can 
convince  yourself  by  distributing  (multiplying)  the  to  each  of  the  terms  of  the 
expression 6 . You can interpret the factorization as the operation of separating all 
the common factors to the terms of the expression. 

  

404141
Example 

Factor 25 10   

What  are  all  the  factors  common  to  25 10 ?  Each  has  at  least  one  .  The 
coefficients  25  and  10  also  have  the  common  factor  5.  5   constitutes  the  greatest 
common  factor  of  25 and  10 .  If  5   is  removed  from  these  terms  (or  better 
factorized),  what  remains  of  25 and  10   ?  Of  25   ,  5   will  remain.  Of  10 ,  2  will 
remain. Consequently,  

25 10 5 5 2    

Example 

Factor the following quadratic functions. 

12 12  

2 12 2 6  

36 6 6 6 1  or‐6 6 1

225 45 15 15 3  

9 16 9 16 3 4 3 4  

Note that in example  5, we used a factorization followed by a difference of squares… 
nothing stops us from using or combining  two factoring methods for one problem. You 
must therefore be at ease with all the methods we suggest here. 

3. Compounded factorization

Consider the quadratic equation  5 6. It is clearly not a problem that can be 
solved by the difference in squares, its form not being suited for this method. Also, you 
will  notice  that  no  common  factor  can  be  found.  Let  us  slightly  adjust  the  quadratic 
function  

5 6 2 3 6

Note that we did not cheat : even though we rewrote the function, its total value  did 
not change. With this new presentation, notice that a factoring can be carried out to the 
pairs  of  terms  2   and  3 6.    x  is  the  common  factor  of  2 ,   and  3  is 
common factor of 3 6. By proceeding to a simple factoring, pair by pair, we obtain  

41
2 3 6 2 3 2 . 

But, this is not all! Now, in this new expression,  2  is a common factor. We can 
therefore  factor  out 2 .  Thus,  from  2 ,  x  remains.  From  3 2 ,    3 
remains. Consequently, 

2 3 2 2 3  

Note  that  we  carried  out  three  consecutive  factorizations  in  order  to  complete  the 
factoring, as the name compounded factorization implies. 

One aspect of the method remains unexplained: why rewrite   under the 
form 2 3 6 instead of  4 6 or                        9
4 6 ? Here is the process allowing us to discover which way to separate the central 
term (multiple of x) : 

Given  , a quadratic function. The term  must be separated to obtain 


the complete factorization. To do this, we must find two numbers, m and n, such that : 

1. the product  c  

2. the sum   

Once  the  numbers  m  and  n  are  found,  replace  the  expression    by 
 and proceed by compounded factorization. 

Example  

Factor the quadratic function  5 6.  

The coefficients are respectively  

Since  0 and  0, we must first separate the central term 5x into two parts. To do 


this, we must find two numbers, m and n, such that  

1. the product  1 6 6 

2. the sum 5 

  

42
Which  two  numbers  have  product  6  and  sum  5?  By  trial  and  error,  you  will  find  that 
2  and 3.  We  use  these  two  numbers  to  substitute  5   by 2 3 .  Thus, 
5 6 2 3 6. Then we use compounded: 

  5 6 2 3 6 2 3 2
2 3  

Example  

Factor the quadratic function 6 17 12. 

The coefficients are respectively 

17

12

Since 0  and  0,  we  must  first  separate  the  central  term, 17   into  two 
parts. To do this, we must find two numbers, m and n, such that  

1.the product  6 12 72  

2. the sum  17

What are the two numbers whose product is 72 and the sum is ‐17 ? You will find that 
8 and  9. Note that the negative signs are very important since we want 
a sum of ‐17. We use these two numbers to substitute  17  by  8 9 . Thus,  

6 17 12 6 8 9 12 

We  can  now  pursue  compounded  factorization,  by  beginning  by  finding  the  common 
factors  to  each  pair  of  terms:  2x  is  a  common  factor  to  6 8   ;  3  is  a  common 
factor to  9 12 (we must generally make sure to factor the negative sign of x if it 
has one). Then we complete the factoring. 

6 17 12 6 8 9 12 2 3 4 3 3 4
3 4 2 3
 

  

43
Example  

Factor the quadratic function 3 10. 

The coefficients are respectively  

10

Since 0 and 0, we must first separate the central term,   into two parts. To 


do this, we need to find two numbers, m and n, such that  

1. the product  3 10 30  

2. the sum  1 

Which  two  numbers  have  product  ‐30  and  sum  ‐1?  The  only  possible  combination  is 
6 and  5. Note that the signs must be chosen to correspond to the criteria 
of product and sum … We use these two numbers to substitute ‐x by ‐6x + 5x. Therefore, 

3 1 12 3 6 5 10 

We  can  now  proceed  by  compounded  factorization,  i.e.  by  first  finding  the  common 
factors to each pair of terms : 

3  is a common factor of 3 6  ; 

5 is a common factor of 5 10. 

We complete the factoring  

 3 10 3 6 5 10 3 2 5 2
2 3 5

  
44
4. Exercises

Factor the following quadratic functions: 

1. 4 25  
2. 3 12  
3. 2 8  
4. 3 7 6  
5. 3 – 28 
6. 12 36  
7. 8 7 1  
8. 4 4 1  
9. 10 3 4  
10. 9 20  

Solutions   

1. 2 5 2 5
2. 3 12 3 4
3. 2 4 2 2 2
4. 3 2 3
5. 7 4
6. 6 6 – 6
7. 8 1 1
8. 2 1 2 1 2 – 1
9. 2 1 5 4
10. 5 4

  

45
3<8Bc4dVT]TD]XeTabXch8

3 SEQUENCES AND SERIES


OF REAL NUMBERS
0DWKHPDWLFVLVWKH4XHHQRI 6FLHQFHVDQGDULWKPHWLF
LVWKH4XHHQRI 0DWKHPDWLFV&)*DXVV

z Introduction Introduction
z Sequences
In this chapter, we shall learn about sequences and series
z Arithmetic Progression (A.P.) of real numbers. Sequences are fundamental mathematical
z Geometric Progression (G.P.) objects with a long history in mathematics. They are tools
z Series for the development of other concepts as well as tools for
mathematization of real life situations.
Let us recall that the letters N and R denote the set
of all positive integers and real numbers respectively.
Let us consider the following real-life situations.
(i) A team of ISRO scientists observes and records the
height of a satellite from the sea level at regular
intervals over a period of time.
(ii) The Railway 0inistry wants to ¿nd out the number of
/HRQDUGR3LVDQR
people using Central railway station in Chennai on a daily
)LERQDFFL
basis and so it records the number of people entering the
(1170-1250)
Central Railway station daily for 180 days.
Italy
(iii) A curious 9th standard student is interested in ¿nding
 )LERQDFFL SOD\HG DQ out all the digits that appear in the decimal part of
LPSRUWDQW UROH LQ UHYLYLQJ DQFLHQW the irrational number  = 2.236067978H and writes
PDWKHPDWLFV +LV QDPH LV down as
NQRZQ WR PRGHUQ PDWKHPDWLFLDQV
2, 3, 6, 0, 6, 7, 9, 7, 8, H .
PDLQO\ EHFDXVH RI  D QXPEHU
(iv) A student interested in ¿nding all positive fractions
VHTXHQFHQDPHGDIWHUKLPNQRZQ
with numerator 1, writes 1, 1 , 1 , 1 , 1 , H .
DV WKH ¶)LERQDFFL QXPEHUV· 2 3 4 5
ZKLFKKHGLGQRWGLVFRYHUEXWXVHG (v) A mathematics teacher writes down the marks of her
DVDQH[DPSOH class according to alphabetical order of the students’
names as 75, 95, 67, 35, 58, 47, 100, 89, 85, 60. .

34 WK6WG0DWKHPDWLFV

46
25
3<8Bc4dVT]TD]XeTabXch

(vi) The same teacher writes down the same data in an ascending order as
35, 47, 58, 60, 67, 75, 85, 89, 95, 100.
In each of the above e[amples, some sets of real numbers have been listed in a speci¿c
order.
1ote that in (iii) and (iv) the arrangements have in¿nite number of terms. In (i), (ii),
(v) and (vi) there are only ¿nite number of terms but in (v) and (vi) the same set of numbers
are written in different order.
Sequences
'HÀQLWLRQ

A sequence of real numbers is an arrangement or a list of real numbers in a speci¿c order.


(i) If a sequence has only ¿nite number of terms, then it is called a ¿nite sequence.
(ii) If a sequence has in¿nitely many terms, then it is called an in¿nite sequence.

n
:e denote a ¿nite sequence as S : a1, a2, a3, H, an or S = {a j} j = 1 and an in¿nite sequence
3
as S : a1, a2, a3, g , an, g or S = {a j} j = 1 where ak denotes the kth term of the sequence. For
example, a denotes the ¿rst term and a denotes the seventh term in the sequence.
Note that in the above examples, (i), (ii), (v) and (vi) are finite sequences, whereas
(iii) and (iv) are in¿nite sequences
Observe that, when we say that a collection of numbers is listed in a sequence, we
mean that the sequence has an identi¿ed ¿rst member, second member, third member and so
on. We have already seen some examples of sequences. Let us consider some more examples
below.
(i) 2, 4, 6, 8, H , 2010. (¿nite number of terms)
(ii) 1,  1, 1,  1, 1,  1, 1, H . (terms just keep oscillating between 1 and  1)
(iii) +, +, +, +, +. (terms are same such sequences are constant sequences)
(iv) 2, 3, 5, 7, 11, 13, 17, 19, 23, H . (list of all prime numbers)
(v) 0.3, 0.33, 0.333, 0.3333, 0.33333, H . (in¿nite number of terms)
(vi) S = " an ,1 where an  1 or 0 according to the outcome head or tail in the nth toss
of a coin.
From the above examples, (i) and (iii) are ¿nite sequences and the other sequences
are in¿nite sequences. One can easily see that some of them, i.e., (i) to (v) have a de¿nite
pattern or rule in the listing and hence we can ¿nd out any term in a particular position in

35

26 47
3<8Bc4dVT]TD]XeTabXch

the sequence. But in (vi), we cannot predict what a particular term is, however, we know it
must be either 1 or 0. Here, we have used the word ‘‘pattern’’ to mean that the nth term of
a sequence is found based on the knowledge of its preceding elements in the sequence. In
general, sequences can be viewed as functions.
Sequences viewed as functions 
n
A ¿nite real sequence aH1, a2, or S = {a } can be viewed as a function
j j=1
af3,: {1, 2, 3, 4, H, n}  R de¿ned by f^ k h, a=n a , k = 1, 2, 3, H, n.
k
3
An in¿nite real sequence a1, a2, a3, g , an, g or S = {a j} j = 1 can be viewed as a
function g  N  R de¿ned by g^ k h = ak,  k  N .
The symbol  means “for all”. If the general term ak of a sequence " ak , is given,
we can construct the whole sequence. Thus, a sequence is a function whose domain is the
set{ 1, 2, 3, H , }of natural numbers, or some subset of the natural numbers and whose
range is a subset of real numbers.
4GOCTMU
A function is not necessarily a sequence. For example, the function f  R  R
given by f (x) = 2x + 1 ,  x  R is not a sequence since the required listing is not
possible. Also, note that the domain of f is not N or a subset { 1, 2, H, n } of N .
Example
Write the ¿rst three terms in a sequence
whose n^n + 1h^2n + 1h th
term is given by
cn = , nN
6
n^n + 1h^2n + 1h
Solution Here, cn = , nN
6
1^1 + 1h^2^1 h + 1h
For n = 1, c  = 1.
6
2^2 + 1h^4 + 1h 2^3 h^5h
For n = 2, c  = = 5.
6 6
3^3 + 1h^7h ^3 h^4h^7h
Finally n = 3, c  = = 14.
6 6
Hence, the ¿rst three terms of the sequence are 1, 5, and 14.
In the above example, we were given a formula for the general term and were able
to ¿nd any particular term directly. In the following example, we shall see another way of
generating a sequence.
Example
Write the ¿rst ¿ve terms of each of the following sequences.
a
(i) a1 =- 1, an = n - 1 , n  1 and  n  N
n+2
(ii) F1 = F2 = 1 and Fn = Fn - 1 + Fn - 2, n = 3, 4, H.

36

27
48
3<8Bc4dVT]TD]XeTabXch

Solution
an - 1
(i) Given a1 =- 1 and an = , n1
n+2
a1
a = =-1
2+2 4
a2 -1
a = = 4 =- 1
3+2 5 20
- 1
a3
a = = 20 =- 1
4+2 6 120
- 1
a4 120 =- 1
a = =
5+2 7 840
A The required terms of the sequence are - 1, - , - 1 , - 1 and - 1 .
1
4 20 120 840

(ii) Given that F1 = F2 = 1 and Fn = Fn - 1 + Fn - 2 , for n = 3, 4, 5, H .


Now, F1 = 1 , F2 = 1
F3 = F2 + F1 = 1 + 1 = 2
F4 = F3 + F2 = 2 + 1 = 3
F5 = F4 + F3 = 3 + 2 = 5
A The ¿rst ¿ve terms of the sequence are 1, 1, 2, 3, 5.

4GOCTMU

The sequence given by F1 = F2 = 1 and Fn = Fn - 1 + Fn - 2,


n = 3, 4, H is called the Fibonacci sequence. Its terms are listed
as 1, 1, 2, 3, 5, 8, 13, 21, 34, H . The Fibonacci sequence occurs in
nature, like the arrangement of seeds in a sunÀower. The number
of spirals in the opposite directions of the seeds in a sunÀower
are consecutive numbers of the Fibonacci sequence.

Exercise 2.1
1. Write the ¿rst three terms of the following sequences whose nth terms are given by
n^n - 2h n+2 ^- 1hn n^n + 2h
(i) an = (ii) cn = ^- 1hn 3 (iii) zn =
3 4
2. Find the indicated terms in each of the sequences whose n terms are given by
th

an = n + 2 ; a7 , a9
n+3
(i) (ii) an = ^- 1hn 2 ^n + 1h ; a5 , a8
2n + 3
n 2
(iii) an = 2n - 3n + 1; a5 , a7. (iv) an = (- 1) (1 - n + n ); a5 , a8
2

37

49
28
3<8Bc4dVT]TD]XeTabXch

3. Find the 18th and 25th terms of the sequence de¿ned by


n (n + 3), if n  N and n is even
an = 22n , if n  N and n is odd.
n +1
4. Find the 13th and 16th terms of the sequence de¿ned by

bn =

2
n, if n  N and n is even
n (n + 2), if n  N and n is odd.
5. Find the ¿rst ¿ve terms of the sequence given by
a1 = 2, a2 = 3 + a1 and an = 2an - 1 + 5 for n  2 .
6. Find the ¿rst six terms of the sequence given by
a1 = a2 = a3 = 1 and an = an - 1 + an - 2 for n   .

Arithmetic sequence or Arithmetic Progression (A.P.)


In this section we shall see some special types of sequences.
'HÀQLWLRQ
A sequence a1, a2, a3, H, an , H is called an arithmetic sequence if
an + 1 = an + d , n  N where d is a constant. Here a is called the first term and
the constant d is called the common difference. An arithmetic sequence is also
called an Arithmetic Progression (A.P.).

Examples
(i) 2, 5, 8, 11, 14, H is an A.P. because a1 = 2 and the common difference d = 3.
(ii)  4,  4,  4,  4, H is an A.P. because a1 =  4 and d = 0.
(iii) 2, 1.5, 1, 0.5, 0, - 0.5, - 1.0, - 1.5, H is an A.P. because a1 = 2 and d =  0.5.

The general form of an A.P.


Let us understand the general form of an A.P. Suppose that a is the ¿rst term and d

is the common difference of an arithmetic sequence {ak} k = 1 . Then, we have
a1 = a and an + 1 = an + d ,  n  N .
For n = 1, 2, 3 we get,
a2 = a1 + d = a + d = a + (2 - 1) d
a3 = a2 + d = (a + d) + d = a + 2d = a + (3 - 1) d
a4 = a3 + d = (a + 2d) + d = a + 3d = a + (4 - 1) d
Following the pattern, we see that the nth term an as
an = an - 1 + d = [a + (n - 2) d] + d = a + (n - 1) d.

38

50
29
3<8Bc4dVT]TD]XeTabXch

Thus , we have an = a + (n - 1) d for every n  N .


So, a typical arithmetic sequence or A.P. looks like
a, a + d, a + 2d, a + 3d, H , a + (n - 1) d, a + nd, H
Also, the formula for the general term of an Arithmetic sequence is of the form
tn = a + (n - 1) d for every n  N .
1RWH
(i) Remember a sequence may also be a ¿nite sequence. So, if an A.P. has only n terms,
then the last term l is given by l = a + ^n - 1h d
(ii) l = a + ^n - 1h d can also be rewritten as n = ` l - a j + 1 . This helps us to ¿nd the
d
number of terms when the ¿rst, the last term and the common difference are given.
(iii) Three consecutive terms of an A.P. may be taken as m - d, m, m + d
(iv) Four consecutive terms of an A.P. may be taken as m - 3d, m - d, m + d, m + 3d
with common difference 2d.
(v) An A.P. remains an A.P. if each of its terms is added or subtracted by a same
constant.
(vi) An A.P. remains an A.P. if each of its terms is multiplied or divided by a non-zero
constant.
Example 2.3
Which of the following sequences are in an A.P.?
(i) 2 , 4 , 6 , H . (ii) 3m - 1, 3m - 3, 3m - 5, H .
3 5 7
Solution
(i) Let tn n E N be the nth term of the given sequence.
A t = 2 , t2 = 4 , t3 = 6
3 5 7
So t2 - t1 = 4 - 2 = 12 - 10 = 2
5 3 15 15

t3 - t2 = 6 - 4 = 30 - 28 = 2
7 5 35 35
Since : t3 - t2 , the given sequence is not an A.P.
t2 - t1 =
(ii) Given 3m - 1, 3m - 3, 3m - 5, H .
Here t = 3m - 1, t2 = 3m - 3, t3 = 3m - 5, H .
A t2 - t1 = (3m - 3) - (3m - 1) =- 2
Also, t3 - t2 = (3m - 5) - (3m - 3) =- 2
Hence, the given sequence is an A.P. with ¿rst term 3m–1 and the common difference –2.

39

51
30
3<8Bc4dVT]TD]XeTabXch

Example 2.4
Find the ¿rst term and common difference of the A.P.
(i) 5, 2, - 1, - 4, H . (ii) 1 , 5 , 7 , 3 , H, 17
2 6 6 2 6
Solution
(i) First term a = 5, and the common difference d = 2 - 5 = - 3 .
(ii) a = 1 and the common difference d = 5 - 1 = 5 - 3 = 1 .
2 6 2 6 3

Example 2.5
Find the smallest positive integer n such that tn of the arithmetic sequence
20, 19 1 , 18 1 , H is negative.?
4 2
Solution Here we have a = 20, d = 19 1 - 20 = - 3 .
4 4
We want to ¿nd the ¿rst positive integer n such that tn   .
This is same as solving a + (n - 1) d  0 for smallest n  N .
That is solving 20 + ^n - 1h`- 3 j  0 for smallest n  N .
4
Now, ^n - 1h`- 3 j  - 20
4
3
(n - 1)   20 ( The inequality is reversed on multiplying both sides by - 1 )
4
A n - 1  20  4 = 80 = 26 2 .
3 3 3
This implies n  26 2 + 1 . That is, n  27 2 = 27.66
3 3
Thus, the smallest positive integer n  N satisfying the inequality is n = 28.

Hence, the 28th term, t28 is the ¿rst negative term of the A.P.

Example 2.6
In a Àower garden, there are 23 rose plants in the ¿rst row, 21 in the second row, 19 in
the third row and so on. There are 5 rose plants in the last row. How many rows are there in
the Àower garden?

Solution Let n be the number of rows in the Àower garden .


The number of rose plants in the 1st, 2nd, 3rd H n th rows are 23, 21, 19, H , 5
respectively.
Now, tk - tk - 1 =- 2 for k = 2, H, n.
Thus, the sequence 23, 21, 19, H , 5 is in an A.P.

40

52
31
3<8Bc4dVT]TD]XeTabXch

We have a = 23, d =- 2, and l = 5 .


A n = - + 1 = 5 - 23 + 1 = 10.
l a
d -2
So, there are 10 rows in the Àower garden.

Example 2.7
If a person joins his work in 2010 with an annual salary of C30,000 and receives an
annual increment of C600 every year, in which year, will his annual salary be C39,000?

Solution Suppose that the person’s annual salary reaches C39,000 in the nth year.
Annual salary of the person in 2010, 2011, 2012, H , [2010 +(n - 1) ] will be
C 30,000, C 30,600, C 31,200, H , C 39000 respectively.

First note that the sequence of salaries form an A.P.


To ¿nd the required number of terms, let us divide each term of the sequence by a
¿xed constant 100. Now, we get the new sequence 300, 306, 312, H , 390.
Here a = 300, d = 6, l = 390.
So, n= l-a +1
d
= 390 - 300 + 1 = 90 + 1 = 16
6 6
Thus, 16th annual salary of the person will be C39,000.
A His annual salary will reach C39,000 in the year 2025.

Example 2.8
Three numbers are in the ratio 2  5  7. If the ¿rst number, the resulting number on the
substraction of 7 from the second number and the third number form an arithmetic sequence,
then ¿nd the numbers.
Solution Let the numbers be 2x, 5x and 7x for some unknown x,( x   )
By the given information, we have that 2x, 5x - 7, 7x are in A.P.
A ^5x - 7h - 2x = 7x - (5x - 7) 3x - 7 = 2x + 7 and so x = 14.
Thus, the required numbers are 28, 70, 98.
Exercise 2.2
1. The ¿rst term of an A.P. is 6 and the common difference is 5. Find the A.P. and its
general term.
2. Find the common difference and 15th term of the A.P. 125, 120, 115, 110, H .
3. Which term of the arithmetic sequence 24, 23 1 , 22 1 , 21 3 , H . is 3?
4 2 4
41

3253
3<8Bc4dVT]TD]XeTabXch

4. Find the 12th term of the A.P. 2, 3 2, 5 2, H .


5. Find the 17th term of the A.P. 4, 9, 14, H .
6. How many terms are there in the following Arithmetic Progressions?
(i) - 1, - 5 , - 2 , H, 10 . (ii) 7, 13, 19, H , 205.
6 3 3
7. If 9th term of an A.P. is zero, prove that its 29th term is double (twice) the 19th term.
8. The 10th and 18th terms of an A.P. are 41 and 73 respectively. Find the 27th term.
9. Find n so that the nth terms of the following two A.P.’s are the same.
1, 7, 13, 19,H and 100, 95, 90, H .
10. How many two digit numbers are divisible by 13?
11. A TV manufacturer has produced 1000 TVs in the seventh year and 1450 TVs in the
tenth year. Assuming that the production increases uniformly by a ¿xed number every
year, ¿nd the number of TVs produced in the ¿rst year and in the 15th year.
12. A man has saved C640 during the ¿rst month, C720 in the second month and C800 in
the third month. If he continues his savings in this sequence, what will be his savings in the
25th month?
13. The sum of three consecutive terms in an A.P. is 6 and their product is –120. Find the
three numbers.
14. Find the three consecutive terms in an A. P. whose sum is 18 and the sum of their
squares is 140.
15. If m times the mth term of an A.P. is equal to n times its nth term, then show that the
(m+n)th term of the A.P. is zero.
16. A person has deposited C25,000 in an investment which yields 14% simple interest
annually. Do these amounts (principal + interest) form an A.P.? If so, determine the
amount of investment after 20 years.
2 2
17. If a, b, c are in A.P. then prove that (a - c) = 4 (b - ac) .

18. If a, b, c are in A.P. then prove that 1 , 1 , 1 are also in A.P.


bc ca ab

19.
2 2 2
If a , b , c are in A.P. then show that 1 , 1 , 1 are also in A.P.
b+c c+a a+b

If a = b = c , x  0, y  0, z  0 and b = ac , then show that 1 , 1 , 1 are in A.P.


x y z 2
20.
x y z

42

54
33
3<8Bc4dVT]TD]XeTabXch

Geometric Sequence or Geometric Progression (G.P.)


'HÀQLWLRQ
A sequence a1, a2, a3, H, an, H is called a geometric sequence if
an + 1 = an r , n  N , where r is a non-zero constant. Here, a is the ¿rst term and
the constant r is called the common ratio. A geometric sequence is also called a
Geometric Progression (G.P.).

Let us consider some examples of geometric sequences.


(i) 3, 6, 12, 24, H .
a
A sequence " an , is a geometric sequence if n + 1 = r  0 , n  N .
an
Now, 6 = 12 = 24 = 2  0 . So the given sequence is a geometric sequence.
3 6 12

(ii) 1 , - 1 , 1 , - 1 ,H .
9 27 81 243
- 1 1 - 1
Here, we have 27 = 81 = 243 = - 1  0 .
1 - 1 1 3
9 27 81
Thus, the given sequence is a geometric sequence.
The general form of a G.P.
Let us derive the general form of a G.P. Suppose that a is the ¿rst term and r is the

common ratio of a geometric sequence {ak} k = 1 . Then, we have
a
a = a and n + 1 = r for n  N .
an
Thus, an + 1 = r an for n  N .
For n = 1, 2, 3 we get,
2-1
a = a1 r = ar = ar
2 3-1
a = a2 r = (ar) r = ar = ar
2 3 4-1
a = a3 r = (ar ) r = ar = ar
Following the pattern, we have
n-2 n-1
an = an - 1 r = (ar ) r = ar .
n-1
Thus, an = ar for every n  N , gives nth term of the G.P.
So, a typical geometric sequence or G.P. looks like
2 3 n-1 n
a, ar, ar , ar , H, ar , ar , H .
Thus , the formula for the general term of a geometric sequence is
n-1
tn = ar , n = 1, 2, 3, H .

43

55
34
3<8Bc4dVT]TD]XeTabXch

Suppose we are given the ¿rst few terms of a sequence, how can we determine if the
given sequence is a geometric sequence or not?
t
If n + 1 = r,  n  N ,where r is a non-zero constant, then " tn , is in G.P.
tn
1RWH
(i) If the ratio of any term other than the ¿rst term to its preceding term of a sequence
is a non-zero constant, then it is a geometric sequence.
(ii) A geometric sequence remains a geometric sequence if each term is multiplied or
divided by a non zero constant.
(iii) Three consecutive terms in a G.P may be taken as a a ar with common ratio r.
r
(iv) Four consecutive terms in a G.P may be taken as a3 , a , ar, ar .
3

 r r
(here, the common ratio is r not r as above)

Example 2.9
Which of the following sequences are geometric sequences
(i) 5, 10, 15, 20, H . (ii) 0.15, 0.015, 0.0015, H . (iii) 7 , 21 , 3 7 , 3 21 , H .
Solution
(i) Considering the ratios of the consecutive terms, we see that 10 =
: 15 .
5 10
Thus, there is no common ratio. Hence it is not a geometric sequence.
(ii) We see that 0.015 = 0.0015 = H = 1 .
0.15 0.015 10
Since the common ratio is 1 , the given sequence is a geometric sequence.
10

(iii) Now, 21 = 3 7 = 3 21 = H = 3 . Thus, the common ratio is .


7 21 3 7
Therefore, the given sequence is a geometric sequence.

Example 2.10
Find the common ratio and the general term of the following geometric sequences.
(i) 2 , 6 , 18 , H . (ii) 0.02, 0.006, 0.0018, H .
5 25 125
Solution
(i) Given sequence is a geometric sequence.
t t
The common ratio is given by r = 2 = 3 = H .
t1 t2
6
Thus, r = 25 = 3 .
2 5
5

44

56
35
3<8Bc4dVT]TD]XeTabXch

The ¿rst term of the sequence is 2 . So, the general term of the sequence is
5
n-1
tn = ar , n = 1, 2, 3, H .
n-1
tn = 2 ` 3 j , n = 1, 2, 3, H
5 5
(ii) The common ratio of the given geometric sequence is
r = 0.006 = 0.3 = 3 .
0.02 10
The ¿rst term of the geometric sequence is 0.02
So, the sequence can be represented by
n-1
tn = (0.02) ` 3 j , n = 1, 2, 3, H
10
Example 2.11
The 4th term of a geometric sequence is 2 and the seventh term is 16 .
3 81
Find the geometric sequence.

Solution Given that t4 = 2 and t7 = 16 .


3 81
n-1
Using the formula tn = ar , n = 1, 2, 3, H . for the general term we have,
t = ar = 2 and t7 = ar = 16 .
3 6
3 81
Note that in order to ¿nd the geometric sequence, we need to ¿nd a and r .
By dividing t by t we obtain,
6
16
t7
= ar3 = 81 = 8 .
t4 ar 2 27
3
3
r = 8 = ` 2 j which implies r = 2 .

Thus,
27 3 3
Now, t = 2 3 2
ar = ` j .
3 3
8
a( ) = . 2 A a = 9.
27 3 4
2 3 n-1 n
Hence, the required geometric sequence is a, ar, ar , ar , H, ar , ar , H
That is, 9 , 9 2 , 9 2 2, H .
4 4`3j 4`3j
Example 2.12
The number of bacteria in a certain culture doubles every hour. If there were 30 bacteria
present in the culture initially, how many bacteria will be present at the end of 14th hour?

Solution Note that the number of bacteria present in the culture doubles at the end of
successive hours.

45

57
36
3<8Bc4dVT]TD]XeTabXch

Number of bacteria present initially in the culture = 30


Number of bacteria present at the end of ¿rst hour = 2 (30)
Number of bacteria present at the end of second hour = 2 (2 (30)) = 30 (22)
Continuing in this way, we see that the number of bacteria present at the end of
every hour forms a G.P. with the common ratio r = 2.
Thus, if tn denotes the number of bacteria after n hours,
n
tn = 30 (2 ) is the general term of the G.P.
14
Hence, the number of bacteria at the end of 14th hour is given by t14 = 30 (2 ) .

Example 2.13
An amount C500 is deposited in a bank which pays annual interest at the rate of 10%
compounded annually. What will be the value of this deposit at the end of 10th year?
Solution
The principal is C500. So, the interest for this principal for one year is 500 ` 10 j = 50 .
100
Thus, the principal for the 2nd year = Principal for 1st year + Interest
= 500 + 500` 10 j = 500 `1 + 10 j
100 100

Now, the interest for the second year = `500 `1 + 10 jj` 10 j .


100 100

So, the principal for the third year = 500`1 + 10 j + 500`1 + 10 j 10


100 100 100
2
= 500`1 + 10 j
100
Continuing in this way we see that
 n-1
the principal for the nth year = 500`1 + 10 j .
100
The amount at the end of (n–1)th year = Principal for the nth year.
Thus, the amount in the account at the end of 10th year
10
= C 500 `1 + 10 j = C 500` 11 j .
10

100 10
4GOCTMU

By using the above method, one can derive a formula for ¿nding the total amount for
compound interest problems. Derive the formula:
n
A = P (1 + i)
where A is the amount, P is the principal, i = r , r is the annual interest rate and
100
n is the number of years.

46

58
37
3<8Bc4dVT]TD]XeTabXch

Example 2.14
The sum of ¿rst three terms of a geometric sequence is 13 and their product is  1.
12
Find the common ratio and the terms.
Solution We may take the ¿rst three terms of the geometric sequence as a a ar .
r
Then, a a ar = 13
r 12
2
1
a ` + 1 + r j = 13 a c r + r + 1 m = 13 (1)
r 12 r 12
Also,
a
` r j^ ah^ar h = - 1

a = -1 A a =- 1
Substituting a =- 1 in (1) we obtain,
2
^- 1hc r + r + 1 m = 13
r 12
2
12r + 12r + 12 = - 13r
2
12r + 25r + 12 = 0
^3r + 4h^4r + 3h = 0

Thus, r = - 4 or - 3
3 4
When r = - and a = – 1, the terms are 3 , –1, 4 .
4
3 4 3
3
When r = - and a = – 1, we get 4 3
, –1, , which is in the reverse order.
4 3 4
Example 2.15
If a b c d are in geometric sequence, then prove that
^b - ch2 + ^c - ah2 + ^d - bh2 = ^a - d h2

Solution Given a b c d are in a geometric sequence.


Let r be the common ratio of the given sequence. Here, the ¿rst term is a.
2 3
Thus, b = ar, c = ar , d = ar
Now, ^b - ch2 + ^c - ah2 + ^d - bh2
2 2 2
= ^ar - ar h + ^ar - ah + ^ar - ar h
2 2 3

= a 6^r - r h + ^r - 1h + ^r - r h @
2 2 2 2 2 3 2

= a 6 r - 2r + r + r - 2r + 1 + r - 2r + r @
2 2 3 4 4 2 6 4 2

2
= a 6 r - 2r + 1 @ = a 6 r - 1 @
2 6 3 2 3

2 2
= ^ar - ah = ^a - ar h = (a - d)
3 3 2

47

59
38
3<8Bc4dVT]TD]XeTabXch8

Exercise 2.3
1. Find out which of the following sequences are geometric sequences. For those
geometric sequences, ¿nd the common ratio.
(i) 0.12, 0.24, 0.48, H . (ii) 0.004, 0.02, 0.1, H . (iii) 1 , 1 , 2 , 4 , H .
2 3 9 27
(iv) 12, 1, 1 , H . (v) 2 , 1 , 1 , H . (vi) 4, - 2, - 1, - 1 , H .
12 2 2 2 2

2. Find the 10th term and common ratio of the geometric sequence 1 , - 1 , 1, - 2, H .
4 2
3. If the 4th and 7th terms of a G.P. are 54 and 1458 respectively, ¿nd the G.P.
4. In a geometric sequence, the ¿rst term is 1 and the sixth term is 1 , ¿nd the G.P.
3 729
5. Which term of the geometric sequence,
(i) 5, 2, 4 , 8 , H , is 128 ? (ii) 1, 2, 4, 8, H , is 1024 ?
5 25 15625
6. If the geometric sequences 162, 54, 18,H . and 2 , 2 , 2 , H have their nth term
81 27 9
equal, ¿nd the value of n.
7. The ¿fth term of a G.P. is 1875. If the ¿rst term is 3, ¿nd the common ratio.
8. The sum of three terms of a geometric sequence is 39 and their product is 1. Find the
10
common ratio and the terms.
9. If the product of three consecutive terms in G.P. is 216 and sum of their products in
pairs is 156, ¿nd them.
10. Find the ¿rst three consecutive terms in G.P. whose sum is 7 and the sum of their
reciprocals is 7
4
11. The sum of the ¿rst three terms of a G.P. is 13 and sum of their squares is 91. Determine
the G.P.
12. If C1000 is deposited in a bank which pays annual interest at the rate of 5% compounded
annually, ¿nd the maturity amount at the end of 12 years .
13. A company purchases an of¿ce copier machine for C50,000. It is estimated that the
copier depreciates in its value at a rate of 15% per year. What will be the value of the
copier after 15 years?
14. If a b c d are in a geometric sequence, then show that
^a - b + ch^b + c + d h = ab + bc + cd.

15. If a b c d are in a G.P., then prove that a + b, b + c, c + d, are also in G.P.

60
39
3<8Bc4dVT]TD]XeTabXch

Series
Let us consider the following problem:
A person joined a job on January 1, 1990 at an annual salary of C25,000 and
received an annual increment of C500 each year. What is the total salary he has received
upto January 1, 2010?
First of all note that his annual salary forms an arithmetic sequence
25000, 25500, 26000, 26500, H, (25000 + 19 (500)) .
To answer the above question, we need to add all of his twenty years salary. That is,
25000 + 25500 + 26000 + 26500 + H + (25000 + 19 (500)) .
So, we need to develop an idea of summing terms of a sequence.

'HÀQLWLRQ
An expression of addition of terms of a sequence is called a series.
If a series consists only a ¿nite number of terms, it is called a ¿nite series.
If a series consists of in¿nite number of terms of a sequence, it is called an in¿nite series.

Consider a sequence S = " an ,n=1


of real numbers. For each n  N we de¿ne the

partial sums by Sn = a1 + a2 + , H + an, n = 1, 2, 3, H . Then {Sn} n = 1 is the sequence of
partial sums of the given sequence " an ,
n=1
.

The ordered pair ^" an , , S 


n = 1 " n ,n = 1h
is called an in¿nite series of terms of the

sequence " an , . The in¿nite series is denoted by a1 + a2 + a3 + H , or simply an
n=1
where the symbol  stands for summation and is pronounced as sigma.
Well, we can easily understand ¿nite series (adding ¿nite number of terms). It is
impossible to add all the terms of an in¿nite sequence by the ordinary addition, since one could
never complete the task. How can we understand (or assign a meaning to) adding in¿nitely
many terms of a sequence? We will learn about this in higher classes in mathematics. For
now we shall focus mostly on ¿nite series.
In this section , we shall study Arithmetic series and Geometric series.

Arithmetic series
An arithmetic series is a series whose terms form an arithmetic sequence.
6XPRI¿UVWn terms of an arithmetic sequence
Consider an arithmetic sequence with ¿rst term a and common difference d
given by a, a + d, a + 2d, ..., a + ^n - 1h d, H .
Let Sn be the sum of ¿rst n terms of the arithmetic sequence.

40
61
3<8Bc4dVT]TD]XeTabXch

Thus, Sn = a + (a + d) + (a + 2d) + H (a + (n - 1) d)
Sn = na + ( d + 2d + 3d + H + (n - 1) d )
= na + d (1 + 2 + 3 + H + (n - 1) )
So, we can simplify this formula if we can ¿nd the sum 1 + 2 + H + (n - 1) .
This is nothing but the sum of the arithmetic sequence 1, 2, 3, H, (n - 1) .
So, ¿rst we ¿nd the sum 1 + 2 + H + (n - 1) below.

Now, let us ¿nd the sum of the ¿rst n positive integers.


Let Sn = 1 + 2 + 3 + H + (n - 2) + (n - 1) + n . (1)
We shall use a small trick to ¿nd the above sum. Note that we can write Sn also as
Sn = n + (n - 1) + (n - 2) + H + 3 + 2 + 1 . (2)
Adding (1) and (2) we obtain,
2Sn = (n + 1) + (n + 1) + H + (n + 1) + (n + 1) . (3)
Now, how many (n + 1) are there on the right hand side of (3)?
There are n terms in each of (1) and (2). We merely added corresponding terms from (1) and (2).
Thus, there must be exactly n such (n + 1) ’s.
Therefore, (3) simpli¿es to 2Sn = n (n + 1) .
Hence, the sum of the ¿rst n positive integers is given by
n (n + 1) n (n + 1)
Sn = . So, 1 + 2 + 3 + H + n = . (4)
2 2
This is a useful formula in ¿nding the sums.
4GOCTMU
The above method was first used by the famous German
mathematician Carl Fredrick Gauss, known as Prince of
Mathematics, to find the sum of positive integers upto 100. This
problem was given to him by his school teacher when he was just ¿ve
years old. When you go to higher studies in mathematics, you will
Carl Fredrick Gauss
learn other methods to arrive at the above formula. (1777 – 1855)

Now, let us go back to summing ¿rst n terms of a general arithmetic sequence.


We have already seen that
Sn = na + [d + 2d + 3d + H + (n - 1) d]
= na + d [1 + 2 + 3 + H + (n - 1)]
n (n - 1)
= na + d using (4)
2
= n [2a + (n - 1) d] (5)
2

50

62
41
3<8Bc4dVT]TD]XeTabXch

Hence, we have
Sn = n [a + (a + (n - 1) d)] = n (¿rst term + last term)
2 
= n (a + l) .
2
The sum Sn of the ¿rst n terms of an arithmetic sequence with ¿rst term a
is given by
(i) Sn = n [2a + (n - 1) d] if the common difference d is given.
2
(ii) Sn = n (a + l) , if the last term l is given.
2

Example 2.16
Find the sum of the arithmetic series 5 + 11 + 17 + H + 95 .

Solution Given that the series 5 + 11 + 17 + H + 95 is an arithmetic series.


Note that a = 5, d = 11 - 5 = 6 , l = 95.
Now, n = l-a +1
d
= 95 - 5 + 1 = 90 + 1 = 16.
6 6
Hence, the sum Sn = n 6l + a @
2
S16 = 16 695 + 5 @ = 8 (100) = 800.
2
Example 2.17
Find the sum of the ¿rst 2n terms of the following series.
2 2 2 2
1 - 2 + 3 - 4 + ... .
2 2 2 2
Solution We want to ¿nd 1 - 2 + 3 - 4 + H to n terms
= 1 - 4 + 9 - 16 + 25 - H to n terms
= ^1 - 4h + ^9 - 16h + ^25 - 36h + H to n terms. (after grouping)
= - 3 + ^- 7h + ^- 11h + H n terms
Now, the above series is in an A.P. with ¿rst term a =- 3 and common difference d =- 4
Therefore, the required sum = n 62a + ^n - 1h d @
2
= n 62^- 3h + ^n - 1h^- 4h@
2
= n 6- 6 - 4n + 4 @ = n 6- 4n - 2 @
2 2
= - 2n ^2n + 1h = - n^2n + 1h .
2

51

63
42
3<8Bc4dVT]TD]XeTabXch

Example 2.18
In an arithmetic series, the sum of ¿rst 14 terms is - 203 and the sum of the next 11
terms is –572. Find the arithmetic series.

Solution Given that S14 = - 203


14 62a + 13d @ = - 203
2
7 62a + 13d @ = - 203
2a + 13d = - 29 . (1)
Also, the sum of the next 11 terms = - 572 .
Now, S25 = S14 + (- 572)
That is, S25 = - 203 - 572 = - 775 .
25 62a + 24d @ = - 775
2
2a + 24d = - 31  2
a + 12d = - 31 (2)
Solving (1) and (2) we get, a = 5 and d =- 3 .
Thus, the required arithmetic series is 5 + ^5 - 3h + ^5 + 2^- 3hh + H .
That is, the series is 5 + 2 - 1 - 4 - 7 - H.

Example 2.19
How many terms of the arithmetic series 24 + 21 + 18 + 15 + H , be taken
continuously so that their sum is – 351.

Solution In the given arithmetic series, a = 24, d =- 3 .


Let us ¿nd n such that Sn = – 351
Now, Sn = n 62a + ^n - 1h d @ = - 351
2
That is, n 62^24h + ^n - 1h^- 3h@ = - 351
2
n 648 - 3n + 3 @ = - 351
2
n^51 - 3nh = - 702
2
n - 17n - 234 = 0
^n - 26h^n + 9h = 0
A n = 26 or n =- 9
Here n, being the number of terms needed, cannot be negative.
Thus, 26 terms are needed to get the sum - 351 .

52

64
43
3<8Bc4dVT]TD]XeTabXch

Example 2.20
Find the sum of all 3 digit natural numbers, which are divisible by 8.
Solution
The three digit natural numbers divisible by 8 are 104, 112, 120, H , 992.
Let Sn denote their sum. That is, Sn = 104 + 112 + 120 + 128 + , H + 992 .
Now, the sequence 104, 112, 120, H , 992 forms an A.P.
Here, a = 104, d = 8 and l = 992.
A n = l - a + 1 = 992 - 104 + 1
d 8
= 888 + 1 = 112.
8
Thus, S112 = 6 a + l @ = 112 6104 + 992 @ = 56 (1096) = 61376 .
n
2 2
Hence, the sum of all three digit numbers, which are divisible by 8 is equal to 61376.
Example 2.21
The measures of the interior angles taken in order of a polygon form an arithmetic
sequence. The least measurement in the sequence is 85D. The greatest measurement is 215D.
Find the number of sides in the given polygon.

Solution Let n denote the number of sides of the polygon.


Now, the measures of interior angles form an arithmetic sequence.
Let the sum of the interior angles of the polygon be
Sn = a + ^a + d h + ^a + 2d h + H l , where a = 85 and l = 215.
We have, Sn = n 6l + a @ (1)
2
We know that the sum of the interior angles of a polygon is (n - 2 )  180 .
0

Thus, Sn = ^n - 2h  180
From (1), we have n 6l + a @ = ^n - 2h  180
2
n 6215 + 85 @ = ^n - 2h  180
2
150n = 180^n - 2h n = 12..
Hence, the number of sides of the polygon is 12.
Exercise 2.4
1. Find the sum of the ¿rst (i) 75 positive integers (ii) 125 natural numbers.
2. Find the sum of the ¿rst 30 terms of an A.P. whose nth term is 3 + 2n .
3. Find the sum of each arithmetic series
(i) 38 + 35 + 32 + H + 2 . (ii) 6 + 5 1 + 4 1 + H 25 terms.
4 2
53

65
44
3<8Bc4dVT]TD]XeTabXch

4. Find the Sn for the following arithmetic series described.


(i) a = 5, n = 30, l = 121 (ii) a = 50, n = 25, d =- 4
2 2 2 2
5. Find the sum of the ¿rst 40 terms of the series 1 - 2 + 3 - 4 + H .
6. In an arithmetic series, the sum of ¿rst 11 terms is 44 and that of the next 11 terms is
55. Find the arithmetic series.
7. In the arithmetic sequence 60, 56, 52, 48,H , starting from the ¿rst term, how many
terms are needed so that their sum is 368?
8. Find the sum of all 3 digit natural numbers, which are divisible by 9.
9. Find the sum of ¿rst 20 terms of the arithmetic series in which 3rd term is 7 and 7th term
is 2 more than three times its 3rd term.
10. Find the sum of all natural numbers between 300 and 500 which are divisible by 11.
11. Solve: 1 + 6 + 11 + 16 + H + x = 148 .
12. Find the sum of all numbers between 100 and 200 which are not divisible by 5.
13. A construction company will be penalised each day for delay in construction of a
bridge. The penalty will be C4000 for the ¿rst day and will increase by C1000 for each
following day. Based on its budget, the company can afford to pay a maximum of
C1,65,000 towards penalty. Find the maximum number of days by which the completion
of work can be delayed
14. A sum of C1000 is deposited every year at 8% simple interest. Calculate the interest
at the end of each year. Do these interest amounts form an A.P.? If so, ¿nd the total
interest at the end of 30 years.
2
15. The sum of ¿rst n terms of a certain series is given as 3n - 2n. Show that the series
is an arithmetic series.
16. If a clock strikes once at 1 o’clock, twice at 2 o’clock and so on, how many times will
it strike in a day?
17. Show that the sum of an arithmetic series whose ¿rst term is a , second term b and the
^a + ch^b + c - 2ah
last term is c is equal to .
2^ b - ah
18. If there are ^2n + 1h terms in an arithmetic series, then prove that the ratio of the sum
of odd terms to the sum of even terms is ^n + 1h : n .
2 2
19. The ratio of the sums of ¿rst m and ¿rst n terms of an arithmetic series is m : n
show that the ratio of the mth and nth terms is ^2m - 1h : ^2n - 1h

54

66
45
3<8Bc4dVT]TD]XeTabXch

20. A gardener plans to construct a trapezoidal shaped structure in his garden. The longer
side of trapezoid needs to start with a row of 97 bricks. Each row must be decreased
by 2 bricks on each end and the construction should stop at 25th row. How many bricks
does he need to buy?

Geometric series
A series is a geometric series if the terms of the series form a geometric sequence.
2 n-1 n
Let a, ar, ar , H, ar , ar , H be a geometric sequence where r = : 0 is the common
ratio. We want to ¿nd the sum of the ¿rst n terms of this sequence.
2 n-1
Let Sn = a + ar + ar + H + ar (1)
If r = 1 , then from (1) it follows that Sn  na .
For r   , using (1) we have
2 n-1 2 3 n
rSn = r (a + ar + ar + H + ar ) = ar + ar + ar + H + ar . (2)
Now subtracting (2) from (1), we get
2 n-1 2 n
Sn  rSn = (a + ar + ar + H + ar ) - (ar + ar + H + ar )
Sn ^1 - r h = a^1 - r h
n

^ n
h
Hence, we have Sn = a 1 - r , since r   .
1-r
The sum of the ¿rst n terms of a geometric series is given by
n n
Sn = r - 1
a (r - 1) a (1 - r )
= , if r  1
1-r
na if r = 1.
where a is the ¿rst term and r is the common ratio.
4GOCTMU
Actually, if - 1  r  1 , then the following formula holds:
a + ar + ar + H + ar + H = a .
2 n
1-r
Note that the sum of in¿nite number of positive numbers may give a ¿nite value.

Example 2.22
Find the sum of the ¿rst 25 terms of the geometric series
16 - 48 + 144 - 432 + H .
^ n
h
Solution Here, a = 16, r = - 48 = - 3  1. Now, Sn = a 1 - r , r  1 .
16 1-r
16^1 - ^- 3h25h 16^1 + 325h
= 4^1 + 3 h .
25
So, we have S25 = =
1 - ^- 3h 4

55

67
46
3<8Bc4dVT]TD]XeTabXch

Example 2.23
Find Sn for each of the geometric series described below:
(i) a = 2, t = 486, n = 6 (ii) a = 2400, r = – 3, n = 5
Solution
(i) Here a = 2, t6 = 486, n=6
5
Now t6 = 2 (r) = 486

r = 243 A r = 3.
a^r - 1h if r  1
n
Now, Sn =
r-1
2 3 - 1h = 36 1 728 .
^ 6
Thus, S = - =
3-1
(ii) Here a = 2400, r =- 3, n = 5
a^r - 1h if r : 1
5
Thus, S = =
r-1
24006^- 3h5 - 1 @
=
^- 3h - 1
= 2400 ^1 + 3 h = 600^1 + 243h = 146400.
5
Hence, S
4

Example 2.24
In the geometric series 2 + 4 + 8 + H , starting from the ¿rst term how many
consecutive terms are needed to yield the sum 1022?

Solution Given the geometric series is 2 + 4 + 8 + H .


Let n be the number of terms required to get the sum.
Here a = 2, r = 2, Sn = 1022 .
To ¿nd n, let us consider
a 6 r - 1 @ if r : 1
n
Sn = =
r-1
n
^2h; 2 - 1 E = 2^2 - 1h .
n
=
2-1
n
But Sn = 1022 and hence 2^2 - 1h = 1022
n
2 - 1 = 511
n 9
 = 512 = 2 . Thus, n = 9.
Example 2.25
The ¿rst term of a geometric series is 375 and the fourth term is 192. Find the common
ratio and the sum of the ¿rst 14 terms.

56

68
47
3<8Bc4dVT]TD]XeTabXch

Solution Let a be the ¿rst term and r be the common ratio of the given G.P.
Given that a = 375, t4 = 192 .
n-1
Now, tn = ar
3 3
A t = 375 r 375 r = 192
r = 192 r = 64
 
375 125
 4 3 4
r =` j r = , which is the required common ratio.
5 5
n
Now, Sn = a ; r - 1 E if r =
:1
r-1
375 8` 4 j - 1 B
14

= (- 1)  5  375 8` 4 j - 1 B
14
Thus, S14 = 5
4 -1 5
5
= ^375h^5h81 - ` 4 j B = 1875 81 - ` 4 j B .
14 14

5 5
1RWH n n
In the above example, one can use Sn = a ; 1 - r E if r =
: 1 instead of Sn = a ; r - 1 E if r =
: 1.
1-r r-1
Example 2.26
A geometric series consists of four terms and has a positive common ratio. The sum
of the ¿rst two terms is 8 and the sum of the last two terms is 72. Find the series.
2 3
Solution Let the sum of the four terms of the geometric series be a + ar + ar + ar and r  
2 3
Given that a + ar = 8 and ar + ar = 72
2 3 2
Now, ar + ar = r (a + ar) = 72
2
r (8) = 72 A r =  
Since r > 0, we have r = 3.
Now, a + ar = 8 a = 2
Thus, the geometric series is 2 + 6 + 18 + 54 .
Example 2.27
Find the sum to n terms of the series 6 + 66 + 666 +H
Solution Note that the given series is not a geometric series.
We need to ¿nd Sn = 6 + 66 + 666 + H to n terms
Sn = 6 (1 + 11 + 111 + H to n terms )
= 6 ^9 + 99 + 999 + H to n termsh (Multiply and divide by 9)
9
2
= 6^10 - 1h + ^100 - 1h + ^1000 - 1h + H to n terms @
3
2 2 3
= [(10 + 10 + 10 + H n terms) - n]
3
n
10 (10 - 1)
Thus, Sn = 2 ; - n E.
3 9
57

69
48
3<8Bc4dVT]TD]XeTabXch

Example 2.28
An organisation plans to plant saplings in 25 streets in a town in such a way that one
sapling for the ¿rst street, two for the second, four for the third, eight for the fourth street and
so on. How many saplings are needed to complete the work?

Solution The number of saplings to be planted for each of the 25 streets in the town
forms a G.P. Let Sn be the total number of saplings needed.
Then, Sn = 1 + 2 + 4 + 8 + 16 + H to 25 terms.
Here, a = 1, r = 2, n = 25
n
Sn = a ; r - 1 E
r-1
6 25 @
S25 = (1) 2 - 1
2-1
25
= 2 -1
25
Thus, the number of saplings to be needed is 2 - 1 .

Exercise 2.5
1. Find the sum of the ¿rst 20 terms of the geometric series 5 + 5 + 5 +H .
2 6 18
2. 1
Find the sum of the ¿rst 27 terms of the geometric series + 1 + 1 +H .
9 27 81
3. Find Sn for each of the geometric series described below.
(i) a = 3, t8 = 384, n = 8 . (ii) a = 5, r = 3 , n = 12 .
4. Find the sum of the following ¿nite series
(i) 1 + 0.1 + 0.01 + 0.001 + H + ^0.1h9 (ii) 1 + 11 + 111 + H to 20 terms.
5. How many consecutive terms starting from the ¿rst term of the series
(i) 3 + 9 + 27 + H would sum to 1092 ? (ii) 2 + 6 + 18 + H would sum to 728 ?
6. The second term of a geometric series is 3 and the common ratio is 4 . Find the sum
5
of ¿rst 23 consecutive terms in the given geometric series.
7. A geometric series consists of four terms and has a positive common ratio. The sum
of the ¿rst two terms is 9 and sum of the last two terms is 36. Find the series.
8. Find the sum of ¿rst n terms of the series
(i) 7 + 77 + 777 + H . (ii) 0.4 + 0.94 + 0.994 + H .
9. Suppose that ¿ve people are ill during the ¿rst week of an epidemic and each sick person
spreads the contagious disease to four other people by the end of the second week and
so on. By the end of 15th week, how many people will be affected by the epidemic?

58

49
70
3<8Bc4dVT]TD]XeTabXch

10. A gardener wanted to reward a boy for his good deeds by giving some mangoes. He
gave the boy two choices. He could either have 1000 mangoes at once or he could
get 1 mango on the ¿rst day, 2 on the second day, 4 on the third day, 8 mangoes on
the fourth day and so on for ten days. Which option should the boy choose to get the
maximum number of mangoes?
11. A geometric series consists of even number of terms. The sum of all terms is 3 times
the sum of odd terms. Find the common ratio.
12. If S1, S2 and S3 are the sum of ¿rst n, 2n and 3n terms of a geometric series respectively,
then prove that S1 ^ S3 - S2h = ^ S2 - S1h2 .
4GOCTMU
The sum of the ¿rst n terms of a geometric series with a = 1 and common ratio x  1,
n
= x - 1, x =
2 n-1
is given by 1 + x + x + H + x : 1.
x-1
Note that the left hand side of the above equation is a special polynomial in x of
degree n - 1 . This formula will be useful in ¿nding the sum of some series.

n n n
Special series  k,  k2 and  k3
k=1 k=1 k=1

We have already used the symbol R for summation.


Let us list out some examples of ¿nite series represented by sigma notation.

Sl. No. Notation Expansion


n n
1. k or j 1+2+3+H+n
k=1 j=1

6
2.  (n - 1) 1+2+3+4+5
n=2

5
3.  (d + 5) 5 + 6 + 7 + 8 + 9 + 10
d=0

n
4.  k2 2 2
1 +2 +3 +H+n
3 2
k=1

10 10
3 61 + 1 + H 10 terms @ = 30.
5. 3 = 3 1
k=1 k=1

n^ n + 1h
We have derived that 1+2+3+H+n = . This can also be obtained
2
n n
using A.P. with a =1 , d = 1 and l = n as Sn = (a + l) = (1 + n) .
2 2
59

71
50
3<8Bc4dVT]TD]XeTabXch

n
Hence, using sigma notation we write it as  k = n (n2+ 1) .
k=1
Let us derive the formulae for
n n n
(i) ^2k - 1h, (ii)  k2 and (iii)  k3 .
k=1 k=1 k=1

Proof:
n
(i) Let us ¿nd ^2k - 1h = 1 + 3 + 5 + H + ^2n - 1h.
k=1

This is an A.P. consisting of n terms with a = 1, d = 2, l = ^2n - 1h .

Sn = n (1 + 2n - 1) = n (Sn = n (a + l) )
2
A
2 2
n
Thus, ^2k - 1h = n2 (1)
k=1

4GOCTMU

1. The formula (1) can also be obtained by the following method


n n n n
2 (n) (n + 1) 
(2k - 1) = 2k - 
1 = 2c km - n =  2

-n = n .
k=1 k=1 k=1 k=1

2
2. From (1), 1 + 3 + 5 + H + l = ` l + 1 j , since l = 2n – 1 n = l + 1 .
2 2

a - b = ^a - bh^a + ab + b h .
3 3 2 2
(ii) We know that
3 2
A k - ^k - 1h3 = k + k^k - 1h + ^k - 1h2 ( take a = k and b = k – 1)
3 2
k - ^k - 1h3 = 3k - 3k + 1 (2)
3 3
When k = 1, 1 - 0 = 3^1 h - 3^1 h + 1 2

3 3
When k = 2, 2 - 1 = 3^2h2 - 3^2h + 1
3 3
When k = 3, 3 - 2 = 3^3 h2 - 3^3 h + 1 . Continuing this, we have
3
when k = n, n - ^n - 1h3 = 3^ nh2 - 3^ nh + 1 .
Adding the above equations corresponding to k = 1, 2, H, n column-wise, we obtain
n = 3 61 + 2 + H + n @ - 3 61 + 2 + H + n @ + n
 2 2 2

Thus, 3 61 + 2 + H + n @ = n + 3 61 + 2 + H + n @ - n
2 2 2 3

 k2 E = n3 + 3n^n2+ 1h - n
n
3;
k=1

n
 k2 = n^n + 1h^2n + 1h
Hence, . (3)
k=1 6

60

72
51
3<8Bc4dVT]TD]XeTabXch

n
(iii)  k3 = 13 + 23 + Hn3
k=1
Let us observe the following pattern.
3
1 = 1 = ^1 h2
3 3
1 +2 = 9 = ^1 + 2h2
3 3 3
1 + 2 + 3 = 36 = ^1 + 2 + 3h2
3 3 3 3
1 + 2 + 3 + 4 = 100 = ^1 + 2 + 3 + 4h2 .
Extending this pattern to n terms, we get
3 3 3 3
1 +2 +3 +H+n = 61 + 2 + 3 + H + n @2
n^ n + 1h 2
=; E
2
n n 2 n^ n + 1h 2
Thus,  k3 =c 
km = ;
2
E . (4)
k=1 k=1

n
(i) The sum of the ¿rst n natural numbers, k = n (n2+ 1) .
k=1 n
The sum of the ¿rst n odd natural numbers, ^2k - 1h = n .
2
(ii)
k=1
(iii) The sum of ¿rst n odd natural numbers (when the last term l is given) is
2
1 + 3 + 5 + H + l = `l + 1j .
2
(iv) The sum of squares of ¿rst n natural numbers,
n
n^n + 1h^2n + 1h
2

k =
6
.
k=1
(v) The sum of cubes of the ¿rst n natural numbers,
n
n^ n + 1h 2
3

k =;
2
E .
k=1

Example 2.29
Find the sum of the following series
(i) 26 + 27 + 28 + H + 60 (ii) 1 + 3 + 5 + H to 25 terms (iii) 31 + 33 + H + 53.
Solution
(i) We have 26 + 27 + 28 + H + 60 = ^1 + 2 + 3 + H + 60h - ^1 + 2 + 3 + H + 25h
60 25
= n - n
1 1
60^60 + 1h 25^25 + 1h
= -
2 2
= (30  61) - (25  13) = 1830 - 325 = 1505.

73
52
3<8Bc4dVT]TD]XeTabXch

(ii) Here, n = 25
n
A 1 + 3 + 5 + H to 25 terms = 25
2
(  (2k - 1) = n2 )
k=1
= 625.

(iii) 31 + 33 + H + 53
= ^1 + 3 + 5 + H + 53h -^1 + 3 + 5 + H + 29h
2 2 2
= ` 53 + 1 j - ` 29 + 1 j ( 1 + 3 + 5 + H + l = `l + 1j )
2 2 2
2 2
= 27 - 15 = 504.
Example 2.30
Find the sum of the following series
2 2 2 2 2 2 2 2
(i) 1 + 2 + 3 + H + 25 (ii) 12 + 13 + 14 + H + 35
2 2 2 2
(iii) 1 + 3 + 5 + H + 51 .
Solution
25
(i)
2
Now, 1 + 2 + 3 + H + 25 =
2 2 2
 n2
1

25^25 + 1h^50 + 1h n
n^n + 1h^2n + 1h
=
6
(  k2 = 6
)
k=1
^25h^26h^51h
=
6
2 2 2 2
A 1 + 2 + 3 + H + 25 = 5525.
2 2 2 2
(ii) Now, 12 + 13 + 14 + H + 35
= ^1 + 2 + 3 + H + 35 h -^1 + 2 + 3 + H + 11 h
2 2 2 2 2 2 2 2

35 11
=  n2 -  n2
1 1

35^35 + 1h^70 + 1h 11^12h^23h


= -
6 6
^35h^36h^71h
^11h^12h^23h
= -
6 6
= 14910 - 506 = 14404 .
2 2 2 2
(iii) Now, 1 + 3 + 5 + H + 51
= ^1 + 2 + 3 + H + 51 h - ^2 + 4 + 6 + H50 h
2 2 2 2 2 2 2 2

51
=  n2 - 22 612 + 22 + 32 + H + 252 @
1

74
53
3<8Bc4dVT]TD]XeTabXch

51 25
=  n2 - 4 n2
1 1

51^51 + 1h^102 + 1h 25^25 + 1h^50 + 1h


= - 4 
6 6
^51h^52h^103h 25^26h^51h
= -4 
6 6
= 45526 - 22100 = 23426.

Example 2.31
Find the sum of the series.
3 3 3 3 3 3 3 3
(i) 1 + 2 + 3 + H + 20 (ii) 11 + 12 + 13 + H + 28
Solution
20
(i)
3 3
1 + 2 + 3 + H20 =
3 3
 n3
1
20^20 + 1h 2
 k3 = ; n^n2+ 1h E .
n 2
=c m using
2 k=1
2
= ` 20  21 j = ^210h2 = 44100.
2
3 3 3
(ii) Next we consider 11 + 12 + H + 28
= ^1 + 2 + 3 + H + 28 h - ^1 + 2 + H + 10 h
3 3 3 3 3 3 3

28 10
=  n3 -  n3
1 1

28^28 + 1h 2 10^10 + 1h 2
=; E -; E
2 2
2 2
= 406 - 55 = (406 + 55) (406 - 55)
= (461)(351) = 161811.
Example 2.32
3 3 3 3
Find the value of k, if 1 + 2 + 3 + H + k = 4356

Solution Note that k is a positive integer.


3 3 3 3
Given that 1 + 2 + 3 + H + k = 4356
k ^ k + 1h 2
c
m = 4356 =6  6  11  11

2
k ^ k + 1h
Taking square root, we get = 66
2
2
k + k - 132 = 0 ^k + 12h^k - 11h = 0
Thus, k = 11 , since k is positive.

75
54
3<8Bc4dVT]TD]XeTabXch

Example 2.33
3 3 3 3
(i) If 1 + 2 + 3 + H + n = 120 , ¿nd 1 + 2 + 3 + H n .
3 3 3 3
(ii) If 1 + 2 + 3 + H + n = 36100, then ¿nd 1 + 2 + 3 + H + n.
Solution
n^ n + 1h
(i) Given 1 + 2 + 3 + H + n = 120 i.e. = 120
2
3 3 3 n^ n + 1h 2 2
A 1 +2 +H+n = c m = 120 = 14400
2
3 3 3 3
(ii) Given 1 + 2 + 3 + H + n = 36100
n^ n + 1h 2
c
m = 36100 =19  19  10  10
2
n^ n + 1h
= 190
2
Thus, 1 + 2 + 3 + H + n = 190.

Example 2.34
Find the total area of 14 squares whose sides are 11 cm, 12 cm, H , 24 cm,
respectively.
2 2 2
Solution The areas of the squares form the series 11 + 12 + H + 24
2 2 2 2
Total area of 14 squares = 11 + 12 + 13 + H + 24
= ^1 + 2 + 3 + H + 24 h -^1 + 2 + 3 + H + 10 h
2 2 2 2 2 2 2 2

24 10
=  n2 -  n2
1 1

24^24 + 1h^48 + 1h 10^10 + 1h^20 + 1h


= -
6 6
^24h^25h^49h ^10h^11h^21h
= -
6 6
= 4900 - 385
= 4515 sq. cm.

Exercise 2.6
1. Find the sum of the following series.
2 2 2 2
(i) 1 + 2 + 3 + H + 45 (ii) 16 + 17 + 18 + H + 25
(iii) 2 + 4 + 6 + H + 100 (iv) 7 + 14 +21 H + 490
2 2 2 2 3 3 3
(v) 5 + 7 + 9 + H + 39 (vi) 16 + 17 + H + 35

76
55
3<8Bc4dVT]TD]XeTabXch <PcWT\PcXRb88

77
3<8Bc4dVT]TD]XeTabXch

78
3<8Bc4dVT]TD]XeTabXch

79
3<8Bc4dVT]TD]XeTabXch

80
3<8Bc4dVT]TD]XeTabXch

81
3<8Bc4dVT]TD]XeTabXch

82
3<8Bc4dVT]TD]XeTabXch

83
3<8Bc4dVT]TD]XeTabXch

84
3<8Bc4dVT]TD]XeTabXch

85
3<8Bc4dVT]TD]XeTabXch

86
3<8Bc4dVT]TD]XeTabXch

87
3<8Bc4dVT]TD]XeTabXch

88
3<8Bc4dVT]TD]XeTabXch

89
3<8Bc4dVT]TD]XeTabXch

90
3<8Bc4dVT]TD]XeTabXch

91
3<8Bc4dVT]TD]XeTabXch

92
3<8Bc4dVT]TD]XeTabXch

93
3<8Bc4dVT]TD]XeTabXch

94
3<8Bc4dVT]TD]XeTabXch

95
3<8Bc4dVT]TD]XeTabXch

96
3<8Bc4dVT]TD]XeTabXch

97
3<8Bc4dVT]TD]XeTabXch

98
3<8Bc4dVT]TD]XeTabXch

99
3<8Bc4dVT]TD]XeTabXch

100
3<8Bc4dVT]TD]XeTabXch

101
3<8Bc4dVT]TD]XeTabXch

102
3<8Bc4dVT]TD]XeTabXch

103
3<8Bc4dVT]TD]XeTabXch

104
3<8Bc4dVT]TD]XeTabXch

105
3<8Bc4dVT]TD]XeTabXch

106
3<8Bc4dVT]TD]XeTabXch

107
3<8Bc4dVT]TD]XeTabXch

108
3<8Bc4dVT]TD]XeTabXch

109
3<8Bc4dVT]TD]XeTabXch

110
3<8Bc4dVT]TD]XeTabXch

111
3<8Bc4dVT]TD]XeTabXch

112
3<8Bc4dVT]TD]XeTabXch

113
3<8Bc4dVT]TD]XeTabXch

114
3<8Bc4dVT]TD]XeTabXch

115
3<8Bc4dVT]TD]XeTabXch

116
3<8Bc4dVT]TD]XeTabXch

117
3<8Bc4dVT]TD]XeTabXch

118
3<8Bc4dVT]TD]XeTabXch

119
3<8Bc4dVT]TD]XeTabXch

120
3<8Bc4dVT]TD]XeTabXch

121
3<8Bc4dVT]TD]XeTabXch

122
3<8Bc4dVT]TD]XeTabXch

123
3<8Bc4dVT]TD]XeTabXch

124
3<8Bc4dVT]TD]XeTabXch

125
3<8Bc4dVT]TD]XeTabXch

126
3<8Bc4dVT]TD]XeTabXch

127
3<8Bc4dVT]TD]XeTabXch

128
3<8Bc4dVT]TD]XeTabXch

129
3<8Bc4dVT]TD]XeTabXch

130
3<8Bc4dVT]TD]XeTabXch

131
3<8Bc4dVT]TD]XeTabXch

132
3<8Bc4dVT]TD]XeTabXch

133
3<8Bc4dVT]TD]XeTabXch

134
3<8Bc4dVT]TD]XeTabXch

135
3<8Bc4dVT]TD]XeTabXch

136
3<8Bc4dVT]TD]XeTabXch

137
3<8Bc4dVT]TD]XeTabXch

138
3<8Bc4dVT]TD]XeTabXch

139
3<8Bc4dVT]TD]XeTabXch

140
3<8Bc4dVT]TD]XeTabXch

141
3<8Bc4dVT]TD]XeTabXch

142
3<8Bc4dVT]TD]XeTabXch

143
3<8Bc4dVT]TD]XeTabXch

144
3<8Bc4dVT]TD]XeTabXch

145
3<8Bc4dVT]TD]XeTabXch

146
3<8Bc4dVT]TD]XeTabXch

147
3<8Bc4dVT]TD]XeTabXch

148
3<8Bc4dVT]TD]XeTabXch

149
3<8Bc4dVT]TD]XeTabXch

150
3<8Bc4dVT]TD]XeTabXch

151
3<8Bc4dVT]TD]XeTabXch

152
3<8Bc4dVT]TD]XeTabXch

153
3<8Bc4dVT]TD]XeTabXch

154
3<8Bc4dVT]TD]XeTabXch

155
3<8Bc4dVT]TD]XeTabXch

156
m e p UNIT 5

STRAND: FINANCE

Simple and Compound


Interest

TEXT

Contents

Section

2.1 Simple Interest

2.2 Compound Interest

2.3 Compound Interest Formula

2.4 Savings: Annual Equivalent Rate (AER)

157
2 Simple and Compound
Interest
2.1 Simple Interest
When money is deposited in a bank or building society account, it commonly attracts
interest; in a similar way, a borrower must normally pay interest on money borrowed.
The rate of interest is usually (but not always) quoted as a rate per cent per year. At the
time of writing a typical rate is 1.5% per annum for money deposited and 1%-2% per
annum for money borrowed. Up-to-date rates are available from finance organisations.
There are two basic ways of calculating the amount of interest paid on money deposited:
simple interest and compound interest.
If simple interest is paid, interest is calculated only on the principal £P, the amount
deposited (the original capital sum). The interest £I payable after one year years at rate
r% per annum is given by the formula
r
I= ×P
100
and the total amount owing can then be calculated by adding I to P.

Worked Example 1
Natasha invests £250 in a building society account. At the end of the year her account is
credited with 2% interest. How much interest had her £250 earned in the year?

Solution
Interest = 2% of £250
2
= × £250
100
= £5

Worked Example 2
Alan invests £140 in an account that pays r% interest. After the first year he receives
£4.20 interest.
What is the value of r, the rate of interest?
Solution
After one year, the amount of interest is given by
r
× £140 = £4.20
100
100 × 4.20
r =
140
420
=
140
= 3
So the interest rate is 3%.
158
1
2.1 CMM Subject Support Strand: Finance Unit 2 Simple and Compound Interest: Text

Exercises
1. Calculate (a) the interest payable and (b) the total amount owing on the following
deposits at simple interest.
(i) £300 borrowed for 5 years at 8% p.a.
(ii) £1000 invested for 4 years at 9.5% p.a.
(iii) £50 borrowed for 2 years at 18% p.a.
(iv) £2500 invested for 6 months at 8.75% p.a. (T = 0.5 years)
(v) £45 000 borrowed for 2 weeks at 15.5% p.a.

The following questions relate to simple interest.


2. What is the actual rate of interest if £4000 deposited for 3 years attracts interest of
£1440?
3. For how long would £500 have to be left in an account paying 4% interest p.a. to
give a balance of £600 ?
4. A school's rich benefactor wants to deposit a certain sum in an account paying
interest at 10.5% so that it will produce interest of £1200 per year, to pay for
scholarships. How much should she deposit?
5. A boy borrows £1.00 from his sister and promises to pay back £1.10 a week later.
What is this as an annual rate of interest?
6. For how long should a depositor leave a sum in a 6.25% p.a. savings account in
order to earn the same amount in interest, assuming the interest is withdrawn each
year?

2.2 Compound Interest


Simple interest is very rarely used in real life: almost all banks and other financial
institutions use compound interest.
This is when interest is added (or compounded) to the principal sum so that interest is
paid on the whole amount. Under this method, if the interest for the first year is left in the
account, the interest for the second year is calculated on the whole amount so far
accumulated.

Worked Example 1
I deposit £250 in a high-earning account paying 9% compound interest and leave it for
three years. What will be the balance on the account at the end of that time?

Solution
Balance after 0 years £250.00
Interest: 9% of £250.00 = £ 22.50
Balance after 1 year: £272.50

Interest: 9% of 272.50 = £ 24.52


Balance after 2 years £297.02

159
2
2.2 CMM Subject Support Strand: Finance Unit 2 Simple and Compound Interest: Text

Interest: 9% of £297.02 = £ 26.73


Balance after 3 years = £323.75

(Note that, for simplicity, all results here are rounded to the nearest penny; computer
calculations are often made to several decimal places.)

Worked Example 2
Jodie invests £1200 in a bank account which pays interest at the rate of 4% per annum.
Calculate the value of her investment after 4 years.

Solution
At an interest rate of 4% per annum, the value of her investment after one year is
4
£1200 + × £1200
100
= 1.04 × £1200
= £1248
After two years, the investment is worth
1.04 × £1248 = £1297.92
and after three years,
1.04 × £1297.92 = £1349.84
At the end of four years, the value of Jodie's investment will be
1.04 × £1349.84 = £1403.83

Exercises
By working from year to year as in the worked example above, calculate the amount
accumulated after three years at compound interest in the following cases.
1. £500 deposited at 10% p.a.

2. £1000 borrowed at 15% p.a.

3. £150 deposited at 6% p.a.

4. £1200 borrowed at 8.5% p.a.

5. £25 000 borrowed at 13.75% p.a.

160
3
2.3 Compound Interest Formula
It should already be clear that for long periods, the year-on-year method of calculating
compound interest is somewhat cumbersome, but fortunately there is a formula.
Suppose the compound interest rate is 9%. The amount at the start of each year is treated
as 100%, and adding 9% to 100% gives 109%. So adding 9% to any amount of money is
equivalent to multiplying that amount by 1.09. Check that
£250.00 × 1.09 = £272.50

£272.50 × 1.09 = £297.02 , and

£297.02 × 1.09 = £323.75


r ⎞
More generally, adding r% to a sum of money corresponds to multiplying by ⎛1 + .
⎝ 100 ⎠
If the money is left untouched for T years, then the original amount £P will be multiplied
r ⎞
by ⎛1 + , so that £A is the total amount at the end of that time,
⎝ 100 ⎠
T
r ⎞
A = P ⎛1 +
⎝ 100 ⎠

(This formula actually works for fractional values of T as well as for whole numbers. The
amount of interest, if it is needed, is calculated by subtracting the principal, £P, from the
total amount.)

Worked Example 1
You borrow £500 for four years and agree to pay 6 12 % compound interest for this period.
What amount will you have to pay back?

Solution
Using the formula,
A = 500 × 1.0654
= 500 × 1.28646...
= 643.233...
So you will have to pay back £643.23, to the nearest penny.
The same formula can be used to calculate the principal sum, the interest rate, or the
length of time, as the following examples show.

Worked Example 2
How much must Sam deposit in a 6% savings account if he wants it to amount to £120
after two years?

161
4
CMM Subject Support Strand: Finance Unit 2 Simple and Compound Interest: Text
2.3

Solution
Using the formula
120 = P × 1.06 2

giving P = 120 ÷ 1.06 2


= 106.799
He must deposit £106.80.

Worked Example 3
The value of a computer depreciates at a rate of 20% per annum. A new computer costs
£1200. What will be its value after
(a) 2 years (b) 6 years (c) 10 years?

Solution
2
20 ⎞
(a) Value = £1200 ⎛1 −
⎝ 100 ⎠
2
4
= £1200 × ⎛ ⎞
⎝ 5⎠
= £768
6
4
(b) Value = £1200 × ⎛ ⎞
⎝ 5⎠
= £314.57
10
4
(c) Value = £1200 × ⎛ ⎞
⎝ 5⎠
= £128.85

Worked Example 4
What rate of interest will allow £350 to grow to £500 in five years?

Solution
From the formula,
5
r ⎞
500 = 350 × ⎛1 +
⎝ 100 ⎠
5
⇒ ⎛1 + r ⎞ = 500 ÷ 350
⎝ 100 ⎠
⇒ = 1.42857...
1
r ⎛ ⎞
⇒ 1+ = 1.42857... ⎜ or 1.42857 5 ⎟
5
100 ⎝ ⎠
= 1.0739
⇒ r = 7.39...
The interest rate is approximately 7.4%

1625
2.3

Worked Example 5
For how long must a sum be deposited in an account paying 14% compound interest in
order to double in value?

Solution
2 P = P × 1.14 T

⇒ 1.14 T = 2
⇒ T log 1.14 = log 2
⇒ T = log 2 ÷ log 1.14

= (0.3010...) ÷ (0.0569...)
= 5.29

The deposit must be left for 5.3 years but as interest is paid yearly, it would have to be
left for 6 years.
5
(Note that after 5 years, the multiplier would be (1.14) ≈ 1.925 but after 6 years
(1.14)6 ≈ 2.195 .)

Exercises
1. Use the formula to calculate the total amount accumulated at compound interest in
the following cases.
(a) £2000 deposited for 5 years at 7% p.a.
(b) £600 borrowed for 8 years at 12% p.a.
(c) £500 deposited for 20 years at 8.25% p.a.
(d) £10 000 borrowed for 6 months at 14% p.a.
(e) £100 deposited for 18 months at 9.5% p.a.

2. Calculate the principal sum which, if deposited at 9.5% compound interest, will grow
to £400 after three years.

3. Calculate the annual rate of compound interest that will allow a principal sum, to
double in value after five years.

4. How long would it take for £1000 to grow to £1500 if deposited at 8% p.a.
compound interest?

5. I borrow £5000 and agree to pay back £6000 after 18 months. What is the annual rate
of compound interest?

6. For how long must you leave an initial deposit of £100 in a 12% savings account to
see it grow to £1000 ?

7. A car costs £9000 and depreciates at a rate of 20% per annum. Find the value of the
car after 3 years.

163
6
CMM Subject Supp
2.3
8. John invests £500 in a building society with interest of 8.4% per annum. Karen
invests £200 at the same rate.
(a) How many years does it take for the value of Karen's investment to become
greater than £300?
(b) How many years does it take for the value of John's investment to become
greater than
(i) £700 (ii) £900?

9. If the rate of inflation were to remain constant at 3%, find the price that a jar of
jam, currently priced at £1.58, would be in 4 years' time.

10. The population of a third world country is 42 million and growing at 2.5% per
annum.
(a) What size will the population be in 3 years' time?
(b) In how many years' time will the population exceed 50 million?

11. The value of a car depreciates at 15% per annum. Bill keeps a car for 4 years and
then sells it.
If the car originally cost £6000, find
(a) its value after 4 years,
(b) the selling price as a percentage of the original value.

2.4 Savings: Annual Equivalent Rate (AER)


The examples in the previous section are all based on interest being paid yearly but, in
reality, interest can be paid biannually, monthly or even daily!
Provided the interest is compounded, this will mean a small increase in the balance of the
account. This is shown in the next example.

Worked Example 1
The nominal interest rate on an account is 6% per annum. After one year, what is the total
value of an initial deposit of £1000 if interest is paid
(a) annually, at the end of the year (b) twice a year, at six-monthly intervals
(c) every month?

Solution
(a) Value = £1000 × 1.06
= £1060.00

(b) If interest is paid every six months, the rate becomes 3% (6% ÷ 2) so that, after
the first 6 months, the value is
£1000 × 1.03 = £1030.00
and after the second six months,
£1030 × 1.03 = £1060.90

164
7
2.4
6%
(c) If paid monthly, the rate is = 0.5% so the monthly multiplier is 1.005 and the
12
value after 12 months is given by

£1000 × (1.005) = £1061.68


12

You can see from the Worked Example above that there is a small gain from having
interest paid more than once per year. Of course, the gain is more significant if the
original investment is a large sum of money, and also if the interest is paid daily!

To compare savings accounts where interest is paid at regular intervals we use the term
Annual Equivalent Rate (AER). Overall, this means that interest can be compounded
more than once in a year depending on the number of times that interest payments are
made. The AER is calculated as
n
i
r = ⎛1 + ⎞ − 1
⎝ n⎠
where r is the AER, n the number of times in a year that interest is paid and i the
'nominal' yearly interest rate.

Worked Example 2
What is the AER for an account with nominal interest rate 6% and interest payments
made each month?

Solution
Here i = 0.06, n = 12 , so
12
0.06 ⎞
r = ⎛1 + −1
⎝ 12 ⎠
= (1 + 0.005) − 1
12

= (1.005) − 1
12

≈ 0.06168
That is, r = 6.17% (as shown by the calculation in part (c) of Worked Example 1).

Worked Example 3
Kate invests £5000 in a savings account that pays interest monthly at a nominal rate of
4.2%.
(a) What is the balance of the account at the end of 3 years?
(b) What is the AER for Karen's investment?

Solution
4.2%
(a) The monthly rate is = 0.35% , so that after 3 years, there have been
12
36 payments of interest, and the balance is

165
8
CMM Subject Support
2.4

£5000 × (1 + 0.0035) ≈ £5000 × 1.1340


36

= £5670.16
(b) The AER formula gives the rate
12
0.042 ⎞
r = ⎛1 + −1
⎝ 12 ⎠

= (1 + 0.0035) − 1
12

= 0.04282

So the AER ≈ 4.28%

Note that, if P is the initial balance of the account and A the balance after n years, then

A = P (1 + R)
n

where R is the APR.

Worked Example 4
If an account with an initial balance of £2500 grows to £3500 after 4 years, what is the
AER?

Solution
If R = AER, then

3500 = 2500 (1 + R)
4

So
3500
(1 + R)4 = = 1.4
2500
and
1
1 + R = (1.4) 4

So 1 + R ≈ 1.087757... , and
R ≈ 0.087757...
Hence AER is 8.78%

£2500 × (1 + 0.0878) = £2500 × (1.0878)


4 4
Check:

= £3500.55

166
9
CMM Subject Support
2.4

Exercises
1. The nominal rate of interest for a savings account is 9% per annum. If £1000 is
invested in the account what will be the balance after one year if
(a) interest is paid once a year, at the end of the year
(b) interest is paid bi-monthly (once every two months)?
2. What is the AER for an account with nominal interest rate 8% and interest payments
made every 3 months?
3. Sara invests £4000 in a savings account that pays interest at a nominal rate of 5.4%,
paid monthly.
(a) What is the AER for this investment?
(b) What is the balance of the account at the end of 5 years?

4. An investment of £2750 grows to £4250 in 5 years. Find the AER for this
investment (as a percentage).

5. Liam invests £3000 in an account that pays interest quarterly (that is, every
3 months) at a rate of 0.6% per month.
(a) What is the AER for this account?
(b) What will be the balance of this account after one year?
(c) What will be the balance of this account after 5 years?

167
10
1. Ratio
Understanding ratio is very closely related to fractions. With ratio comparisons are made between
equal sized parts/units. Ratios use the symbol “ : “ to separate the quantities being compared.
For example, 1:3 means 1 unit to 3 units, and the units are the same size. Whilst ratio can be
1
expressed as a fraction, the ratio 1:3 is NOT the same as , as the rectangle below illustrates
3

The rectangle is 1 part grey to 3 parts white. Ratio is 1:3 (4 parts/units)


The rectangle is a total of 4 parts, and therefore, the grey part
1 1
represented symbolically is and not
4 3

AN EXAMPLE TO BEGIN:

My two-stroke mower requires petrol and oil mixed to a ratio of 1:25. This means that I add one part
oil to 25 parts petrol. No matter what measuring device I use, the ratio must stay the same. So if I
add 200mL of oil to my tin, I add 200mL x 25 = 5000mL of petrol. Note that the total volume (oil and
petrol combined) = 5200mL which can be converted to 5.2 litres. Ratio relationships are
multiplicative.
Mathematically: 1:25 is the same as
2:50 is the same as
100:2500 and so on.
To verbalise we say 1 is to 25, as 2 is to 50, as 100 is to 2500, and so on……

Ratios in the real world:

 House plans 1cm:1m = 1:100


 Map scales 1:200 000
 Circles C:D is as π: 1
 Golden ratio 𝜑𝜑: 1 as is 1.618:1often used in art and design as
shown right with the golden rectangle. This is also the rectangle
ratio used for credit cards.
(Image from http://www.flickr.com/photos/yorkjason/1456384155/)

Key ideas:

Part-to-part ratio relationships


 Comparison of two quantities (e.g. number of boys to girls)
 A ratio is a way of comparing amounts
 A ratio shows the number of times an amount is
contained in another, or how much bigger one
amount is than another
 The two numbers are both parts of the whole
 If I need to mix some cement, then I could add
two parts cement to four parts sand. Hence the
ratio 2:4 (6 parts in total). The written expression
is important; 4:2 would give a different mix.

168
 Ratios are written to their simplest form. In the figure to
the right we have 15 red dots and five green dots. The
ratio is 15:5; however, this can be reduced to the simplest
form as we do with fractions. The ratio as we can see in
the graphic is also 3:1, if you look at the relationship of the
numbers 15:5 as is 3:1, we can see that 3 is multiplied by 5
to get 15 and so is the one to get five.
 We can also think of the ratio as being a part of the whole
100%, so in the instance above, we would have 75%:25%
(each being a part of 100%). If we had 100 dots, 75 of
them would be red and 25 of them would be green.
15:5 is as 3:1
Part-to-whole ratio relationship: (e.g. boys to class)

 One quantity is part of another; it is a fraction of the whole.


 For example, there are 12 boys as part of 30, which is written 12:30

1. Your Turn:
Write down as many things as you can about the ratio of the table above.
a. What is the part to part ratio?

b. What is the part to whole ratio?

c. What is the ratio of part to part in the simplest form?

d. What is the ratio of part to whole in the simplest form?

e. What is the ratio as a percentage for part to part?

169
2. Direct and Indirect Proportion

Indirect proportion – sometimes called ‘inverse proportion’

When a variable is multiplied by a number and the other variable


Num. of 8 4 2 1
is divided by the same number – they are said to be in indirect people
proportion. For example, it takes 8 people ½ hour to mow the
lawn, 4 people take one hour, 2 people 2 hours, and 1 person 4 Time in 1/2 1 2 4
hours
hours.

Direct proportion

When one variable changes, the other changes in a related way; the change is constant and
multiplicative. For example, if we look at the ratios, 1:25 and 2:50 we multiply both variables by two.

𝑠𝑠𝑠𝑠𝑠𝑠𝑠𝑠𝑠𝑠 𝑤𝑤𝑤𝑤𝑤𝑤ℎ 1: 25, 1 × 2 = 2 𝑎𝑎𝑎𝑎𝑎𝑎 25 × 2 = 50 𝑡𝑡𝑡𝑡 𝑔𝑔𝑔𝑔𝑔𝑔 2: 50.

Let’s look at this again in a practical situation: A man plants two trees every ten minutes, four trees
every 20 mins, six trees every 30 mins… 2:10, 4:20, …

We can measure the height of a tree on a sunny day without having


to go up a ladder. We simply need to measure the height of a stick, in
the same vicinity, and the shadows that they both cast. Here we are
dealing with the cross products (meaning the multiplicative
relationship) of 2 equivalent ratios.

For example, if a tree shadow is 13m, and we measure a shadow of a


30cm upright ruler in the same vicinity we can work out how tall the
tree is without climbing a ladder! But first we will convert to the same
unit. The stick is 30cm and its shadow is 50cm, thus 13m for the
shadow of the tree is converted to 1300cm.

30cm
50cm 𝒙𝒙
13m (1300cm)

30 𝑥𝑥
If we know that 30 𝑖𝑖𝑖𝑖 𝑡𝑡𝑡𝑡 50 is as 𝑥𝑥 𝑖𝑖𝑖𝑖 𝑡𝑡𝑡𝑡 1300, then we can write 𝑖𝑖𝑖𝑖 𝑒𝑒𝑒𝑒𝑒𝑒𝑒𝑒𝑒𝑒𝑒𝑒𝑒𝑒𝑒𝑒𝑒𝑒𝑒𝑒 𝑡𝑡𝑡𝑡
50 1300

Next we can use a method of cross multiplication because 𝑥𝑥 × 50 𝑖𝑖𝑖𝑖 𝑒𝑒𝑒𝑒𝑒𝑒𝑒𝑒𝑒𝑒𝑒𝑒𝑒𝑒𝑒𝑒𝑒𝑒𝑒𝑒 𝑡𝑡𝑡𝑡 30 × 1300

170
30 𝑥𝑥
50 1300

𝑥𝑥 × 50 = 30 × 1300 we apply the rule that what we do to one side we do to the other.
30×1300
𝑥𝑥 × 50÷50 =
50
30×1300÷50 30×26
𝑥𝑥 = =
50÷50 1

𝑥𝑥 = 780
Therefore, 30:50 is the same as 780:1300 so the tree is 780cm or 7.8m tall.
Let’s check 30:50 is the same as 3:5, this is also in the same proportions as 780:1300.
The change has been constant and multiplicative and one of direct proportion.

Without realising it, we often use ratio and direct proportion, and hence, multiplicative thinking
daily. For instance, when we cook we may be required to convert measurements. We do this
mentally without realising the mathematical thinking we have just so expertly applied …

EXAMPLE PROBLEM:

Making biscuits uses proportion. My recipe states that to make 50 biscuits, I need
5 cups of flour 1 tin of condensed milk
½ a cup of sugar 150g of choc chips.
How much of each ingredient will I need if I require 150 biscuits?
To calculate we need to find the ratio. If the recipe makes 50 and we need 150 we triple the recipe
(150 ÷50 = 3 ) 50:150 is an equivalent ratio to 1:3 that is they are in the same proportions
Therefore, 5 x 3 cups of flour = 15 cups,
1 3 3
× = = 1 ½ cups of sugar.
2 1 2
1 × 3 = 3 tins of condensed milk and
150g x 3 = 450g of choc chips.

2. Your Turn:
Write each ratio in its simplest form:

a. $450 : $600 e. 4.5kg : 9.0g

b. 25mm : 1.00m f. 250mL : 2.00L

c. 2.5cm : 5.00km g. 400m : 80mm

d. 30sec : 1hr h. 50kg : 1.20t

i. A standard white bread loaf recipe How much flour is needed if:
might be: 100 parts flour i. I use 50 grams of salt?
65 parts warm water ii. I use 13 cups of water?
2 parts salt
1.5 parts yeast

171
3. Rate
A rate is a numerical comparison between two different kinds of quantities. A rate must have units,
quantity per quantity. For instance, we can buy coffee at $9.80 for every kilogram. The two variables
are money and kilograms. The term ‘per’ is a term used in exchange for the phrase ‘for every,’ so we
buy coffee at $9.80 per kg. Other examples:
• Km per hour, km/hr
• Food prices: $ per weight
• Wages: $ per hour
• Rate can be expressed using a line graph

EXAMPLE PROBLEM 1:

An intravenous line has been inserted in a patient. Fluid is


600
being delivered at a rate of 42mL/h. How much fluid will the
500
patient receive in: 400

Fluid mLs
• 2 hours? 42mL × 2h = 84mL 300
• 8 hours? 42mL × 8h = 336mL 200
• 12 hours? 42mL × 12h = 504mL 100
0
A rate that is constant is related to a linear graph. The line
0 5 10 15
that passes though the origin has a gradient which we call
hours
the rise and run, written as rise: run and more commonly as
𝑟𝑟𝑖𝑖𝑠𝑠𝑒𝑒/ (𝑟𝑟𝑢𝑢𝑛𝑛) for this graph 420/10 ∴ a positive gradient of 42.
This concept will be covered more in later modules.

EXAMPLE PROBLEM 2:

William needs to drive to a neighbouring city. Part of the journey will cover 308km of winding road.
He travelled 208km in two hours at an average speed over that distance. If he continues to travel at
that average speed, how long will it take him to complete the 308km section of road?

Step One: Gather the facts/information


What is the distance to be covered? 308km
What is the distance covered over what time? 112 km in 2hrs
Step Two: Find a solution map; a formula to apply to solve the problem
𝑑𝑑𝑑𝑑𝑑𝑑𝑑𝑑𝑑𝑑𝑑𝑑𝑑𝑑𝑑𝑑
1) = 𝑎𝑎𝑎𝑎𝑎𝑎𝑎𝑎𝑎𝑎𝑎𝑎𝑎𝑎 𝑠𝑠𝑠𝑠𝑠𝑠𝑠𝑠𝑠𝑠
𝑡𝑡𝑡𝑡𝑡𝑡𝑡𝑡
𝑑𝑑𝑑𝑑𝑑𝑑𝑑𝑑𝑑𝑑𝑑𝑑𝑑𝑑𝑑𝑑 𝑐𝑐𝑐𝑐𝑐𝑐𝑐𝑐𝑐𝑐𝑐𝑐𝑐𝑐
2) How to calculate the time taken: = 𝑡𝑡𝑡𝑡𝑡𝑡𝑡𝑡
𝑠𝑠𝑠𝑠𝑠𝑠𝑠𝑠𝑠𝑠
Step Three: Solve
112
1) Thus = 56 ∴ the average speed is 56km/hr
2

308 11 1
2) Thus = =5
56 2 2
∴ 𝐼𝐼𝐼𝐼 𝑤𝑤𝑤𝑤𝑤𝑤𝑤𝑤 𝑡𝑡𝑡𝑡𝑡𝑡𝑡𝑡 𝑊𝑊𝑊𝑊𝑊𝑊𝑊𝑊𝑊𝑊𝑊𝑊𝑊𝑊 𝑓𝑓𝑓𝑓𝑓𝑓𝑓𝑓 ℎ𝑜𝑜𝑜𝑜𝑜𝑜𝑜𝑜 𝑎𝑎𝑎𝑎𝑎𝑎 𝑡𝑡ℎ𝑟𝑟𝑟𝑟𝑟𝑟𝑟𝑟 𝑚𝑚𝑚𝑚𝑚𝑚𝑚𝑚𝑚𝑚𝑚𝑚𝑚𝑚 𝑡𝑡𝑡𝑡 𝑡𝑡𝑡𝑡𝑡𝑡𝑡𝑡𝑡𝑡𝑡𝑡 308𝑘𝑘𝑘𝑘.
Step Four: Check by working backwards
56 × 5.5 = 308

172
3. Your Turn:
Use the steps above to solve the problem below. It is good practice to apply this way of working
because it will help you to structure your mathematical thinking and reasoning in the future.

a. Sonia travels a distance of 156km and then turns right to travel a further 120km. The journey
takes her six hours. What is the average speed of her journey?

Step One:

Step Two:

Step Three:

Step Four:

4. Nursing Examples
Nurses will calculate IV rates: drops per minute (dpm)

Some information is required first:


• The total volume to be given, which is often written on the prescription in mLs.
• The time over which the volume is to be given, often in minutes
• The drop factor (determined by the administration set). This means how many drops per mL,
which are commonly 15, 20 or 60drops/mL
𝑡𝑡𝑡𝑡𝑡𝑡𝑡𝑡𝑡𝑡 𝑣𝑣𝑣𝑣𝑣𝑣𝑣𝑣𝑣𝑣𝑣𝑣 𝑡𝑡𝑡𝑡 𝑏𝑏𝑏𝑏 𝑔𝑔𝑔𝑔𝑔𝑔𝑔𝑔𝑔𝑔 (𝑖𝑖𝑖𝑖 𝑚𝑚𝑚𝑚𝑚𝑚) 𝑑𝑑𝑑𝑑𝑑𝑑𝑑𝑑 𝑓𝑓𝑓𝑓𝑓𝑓𝑓𝑓𝑓𝑓𝑓𝑓
× = 𝑑𝑑𝑑𝑑𝑑𝑑𝑑𝑑𝑑𝑑 𝑝𝑝𝑝𝑝𝑝𝑝 𝑚𝑚𝑚𝑚𝑚𝑚𝑚𝑚𝑚𝑚𝑚𝑚
𝑡𝑡𝑡𝑡𝑡𝑡𝑡𝑡 (𝑖𝑖𝑖𝑖 𝑚𝑚𝑚𝑚𝑚𝑚𝑚𝑚𝑚𝑚𝑚𝑚𝑚𝑚) 1

𝑥𝑥
To generalise: what we are doing is dividing two variables, then we multiply that by the constant,
𝑦𝑦
𝑥𝑥 20
which is the drop factor, to get 𝑘𝑘 (in this case dpm); therefore, × = 𝑘𝑘
𝑦𝑦 1

173
EXAMPLE PROBLEM :

What is the IV rate for 1500mLs to be given over 10 hours with a drop factor of 20?

1500mLs 20
× = 𝑘𝑘 dpm
10hrs 1

Because we are looking for drops per minute we convert the hours to minutes (10 × 60 =
600mins)

1500mLs 20
× = 50dpm
600mins 1

In summary, this formula involves direct proportional thinking and reasoning. The drop factor rate is
the constant, drops/mL. Hence, we are looking at the relationship of the time and the total to be
given at a constant rate of 15, 20 or 60 drops per mL. Because the variables of volume and time
differ and the drop factor rate remains constant, we can calculate how many drops per minute
(dpm) will fall.

Drug Dosage

Another example might be calculating the dosage of a drug to be given based on an individual’s
weight, which involves a rate, drug per kilo. We would follow the same direct proportional thinking
𝑥𝑥 𝑏𝑏𝑏𝑏𝑏𝑏𝑏𝑏 𝑤𝑤𝑤𝑤𝑤𝑤𝑤𝑤ℎ𝑡𝑡
as above: × = 𝑘𝑘
𝑦𝑦 1

We would require some information to begin. It is important to always start with what we know,
that is what information is on hand. For instance, what is the amount of drug (𝑥𝑥) ÷ (𝑦𝑦)per kilo, and
then, what is the body weight?

Let’s investigate; if a 60kg individual is required to take a drug that is administered at a rate of 10mg
𝑥𝑥
per 1kg, then we can apply the formula: × = 𝑘𝑘
𝑦𝑦 1
10mg 60kg
Let’s substitute the variables with values: × = 𝑘𝑘 mg
1kg 1

10mg 60kg 600mg


∴ × = = 600mg
1kg 1 1

So then what happens if the stock is given at 250mg/5mL? Let’s apply the same thinking…
𝑥𝑥 5mL 600mg 5mL 600mg
× = 𝑘𝑘 Now we substitute with values: × = 𝑘𝑘 mL × 1 = 𝑘𝑘 mL
𝑦𝑦 1 250mg 1 250mg

5mL 600÷25 5 × 24
× = = 𝑘𝑘 mL
250÷25 1 10

5÷5 × 24 24
= = 12 mL
10÷5 2

When you work the equations as above, you can apply the cancel out method. Always remember
that what you do to one side, you do to the other. These are examples to investigate and there are
many ways to work mathematically. With practise you will find your own method.

174
4. Your Turn:
Apply your understanding of direct proportional thinking to solve the following:

𝑥𝑥 𝑐𝑐𝑐𝑐𝑐𝑐𝑐𝑐𝑐𝑐𝑐𝑐𝑐𝑐𝑐𝑐
× = 𝑘𝑘
𝑦𝑦 1

a. A patient is prescribed 150mg of soluble aspirin. We only have 300mg tablets on hand. How
many tablets should be given?

b. A solution contains fluoxetine 20mg/5mL. How many milligrams of fluoxetine are in 40mL of
solution?

c. A stock has the strength of 5000units per mL. What volume must be drawn up into an
injection to give 6500units?

d. An intravenous line has been inserted in a patient. The total volume to be given is 1200mLs
over 5hours at a drop factor of 15drops/mL. How many drops per minute will the patient
receive?

e. Penicillin syrup contains 200mg of penicillin in 5mL of water. If a patient requires 300mg of
penicillin how much water will be required to make the syrup?

175
5. Percent
Key ideas:

• The concept of percentage is an extension of the material we have covered about fractions
and ratio. To allow easy comparisons of fractions we need to use the same denominator. As
such, percentages use 100 as the denominator. In other words, the ‘whole’ is divided into
27
100 equal parts. The word “per cent” means per 100. Therefore 27% is .
100

• To use percentage in a calculation, the simple mathematical procedure is modelled below


where we write the percentage as a fraction with a denominator of 100. For example: 25%
25 25 40
is thus to calculate 25%of 40, we could approach it as × = 10
100 100 1

• Percentages are most commonly used to compare parts of an original. For instance, the
phrase ‘30% off sale!’ indicates that whatever the original price, the new price will be 30%
less.

• However, percentages are rarely as simple as 23% of 60. Percentages are more commonly
used as part of a more complex question. Often the question is, “How much is left?” or “How
much was the original?”

EXAMPLE PROBLEMS:
I am purchasing a new iPod for $300; at present, they are reduced by 15%.
How much will my iPod cost?
Method one:
If for every $100.00 spent I save $15.00, then there are three hundreds in 300, so
15x3 is 45, [think (10x3) + (5x3)]
Step 1: SIMPLE PERCENTAGE
15% of 300 is $45.00,
Step 2: DIFFERENCE
300-45=255 therefore, the iPod will cost $255.00
Method two: 17517
Step 1: SIMPLE PERCENTAGE
If 15% is the same as 15/100, which is also the same as 0.15, then 300 x 0.15 is 45.
Step 2: DIFFERENCE
300 – 45 is 255 therefore, the iPod will cost $255.00
Method three:
Step 1: SIMPLE PERCENTAGE
If 15% is 10% + 5%, then 10% of 300 is 30, half of 30 is 15 (5% of 300)
So 30 + 15 𝑖𝑖𝑖𝑖 45.
Step 2: DIFFERENCE: 300 − 45 = 255 therefore, the iPod will cost $255.00

176
An advertisement at the chicken shop states that on Tuesday everything is 22% off. If chicken
breasts are normally $9.99 per kilo, what is the new per kilo price?
Step 1: SIMPLE PERCENTAGE:
22 9.99
× = 2.20
100 1
Step 2: DIFFERENCE: Since the price is 22% cheaper, $2.20 is subtracted from the original.
9.99 – 2.20 = $7.79

For the new financial year, you have been given an automatic 5% pay rise. If you were originally
on $17.60 per hour, what is your new rate?
5 17.60
Step 1: SIMPLE PERCENTAGE: × = 0.88
100 1
Step 2: DIFFERENCE: Since it is a 5% pay RISE, the $0.88 is added to the original.
17.60 + 0.88 = $18.48

Or, we could simplify this into one step:


If your new salary is the existing salary (100%) plus the increase (5%), so 105%, then you
could calculate your new salary by finding 105% of the existing salary. Remembering that
105% is 1.05 as a decimal, we simply multiply 17.60 by 1.05, therefore, 17.60 x 1.05 = $18.48

A new dress is now $237 reduced from $410. What is the percentage difference? As you can see,
the problem is in reverse, so we approach it in reverse!
Step 1: DIFFERENCE
Since it is a discount the difference between the two is the discount.
$410 – $237 = $173
Step 2: PERCENTAGE: now we need to calculate what percentage of $410 was $173, and so
𝑥𝑥 410
we can use this equation: × = 173
100 1
𝑥𝑥 410÷410 173÷410
We can rearrange the problem in steps: × = this step involved dividing
100 1 1
𝑥𝑥 173
410 from both sides to get =
100 410
Next we work to get the 𝑥𝑥 on its own, so we multiply both sides by 100. Now we have
173 100
𝑥𝑥 = ×
410 1
Next we solve, (173 × 100) ÷ 410 = 42.20 .
∴ The percentage difference was 42.2%.
Check: 42.2% of $410 is $173, $410 − $173 = $237, the cost of the dress was $237.00.

We want to buy some shoes that are reduced by 15%. The existing price is $200.00 and the new
price will be 15% less. We can work this in one easy step (as above):

First, we can deduct 15% from 100% giving us 85%. Then 85% as a decimal is 0.85, so we
simply use 200 x 0.85 = 170, therefore, our new price is $170.

177
Now for something a bit trickier:
What if we were to add 10%GST to an item that costs $5.50.
We can do this in one step, again converting percentage to decimals: 5.50 × 1.10 = $6.05
What if we wanted to calculate the original cost. Can we simply subtract 10% from $6.05?
10 6.05
6.05 × 0.9 = $5.45 or × = 0.61 (6.05 − 0.605 = $5.45)Which is close to $5.50;
100 1
but not exact, thus it is incorrect. So what do we do?
We need to think that if 10% was added, then $6.05 is 110% so…
To calculate we write: If $6.05 is 110%, then what was 100%?
6.05 100 605
∴ × = = 5.5
110 1 110
Now we are back to the original cost of $5.50. 

Let’s try another: if we have a markup of 25% on a $300 shirt, the retail price will be $375
300 × 1.25 = 375 (using the one step method from above)
Now let’s get back to the original cost. $375 now represents 125%
What is 100% ?
375 100
× = (divide both sides by 25)
125 1
15÷5 100 3 100
× = × = 300
5÷5 1 1 1

5. Your Turn:
a. I want to buy some shoes that are reduced by 35%. Their original price is marked at $200.
How much will I pay for the discounted shoes?

b. You have received a 6% increase on your weekly pay. If you originally received $512 per
week, what is your new weekly wage?

c. In 2012, an ornithologist conducted a survey of brolgas sighted in a nearby wetland. She


counted 24 individuals. In 2013, she counted 30 individuals. By what percentage did the
population of brolgas increase?

d. Fiona scored 24 out of 30 on her first maths test. On her second test, she scored 22 out of
30. By what percentage did Fiona’s maths results change?

e. A computer screen was $299 but is on special for $249. What is the percentage discount?

178
6. Combine Concepts: A Word Problem
What I need to get on my final exam?

Grade (%) Weight (%)


Assessment 1 30.0% 10%
Assessment 2 61.0% 15%
Assessment 3 73.2% 30%
Assessment 4 51.2% 5%
Final Exam 40%

Final Grade Overall Percentage Needed


High Distinction 100 - 85%
Distinction 84 - 75%
Credit 74 - 65%
Pass 64 - 50%
Fail 49 - 0%

1. How much does each of my assessments contribute to my overall percentage,


which determines my final grade?

Calculation Overall %
Assessment 1 This assessment contributes a maximum of 10% to my
overall percentage and I scored 30.0% of those 10%, so
30.0 ÷ 100 x 10 = 3
𝟑𝟑𝟑𝟑 𝟏𝟏𝟏𝟏
× 𝟏𝟏 = 𝟑𝟑 3.0 %
𝟏𝟏𝟏𝟏𝟏𝟏

Assessment 2 This assessment contributes a maximum of 15% to my


overall percentage and I scored 61.0% of those 15%, so

Assessment 3

Assessment 4

Total

Check: Your total should be 36.67%

179
2. What do I need to score on the final exam to get a P, C, or a D? Can I
still get a HD?
Calculation Required
score
P For a Pass, I need to get at least 50% overall. I already have
36.67%, so the final exam needs to contribute 50 - 36.67 = 13.33
The exam contributes a maximum of 40% to my overall grade,
but I only need to get 13.33%, so how many percent of 40% is
13.33%?
? ÷ 100 x 40 = 13.33
? = 13.33 ÷ 40% x 100 33.33%
? = 33.33
C For a Credit, I need to get at least 65% overall. I already have
36.67%, so the final exam needs to contribute:

The exam contributes a maximum of 40% to my overall grade,


but I only need to get , so:

HD

Note: In most subjects, you need to score a certain percentage on the exam to pass
the subject regardless of your previous results. In some cases, this may be up to 50%.
Check your subject outline!

180
7. Answers
1) a. 12:18 b. 12:30 boys : students or 18:30 girls : students
c. 2:3 d. 2:5 boys : students or 3:5 girls : students e. 40%:60%

2) a. 3:4 b. 1:40 c. 1:200,000 d. 1:120


e. 500:1 f. 1:8 g. 5000:1 h. 1:24
i. i) 2500grams ii) 20cups

3) a. Step one: Distance covered: 156km + 120km = 276km. Time: six hours.
𝑑𝑑𝑑𝑑𝑑𝑑𝑑𝑑𝑑𝑑𝑑𝑑𝑑𝑑𝑑𝑑 𝑐𝑐𝑐𝑐𝑐𝑐𝑐𝑐𝑐𝑐𝑐𝑐𝑐𝑐
Step Two: Solution map: = 𝑠𝑠𝑠𝑠𝑠𝑠𝑠𝑠𝑠𝑠
𝑡𝑡𝑡𝑡𝑡𝑡𝑡𝑡
276
Step Three: solve: = 46 ∴ 46𝑘𝑘𝑘𝑘/ℎ𝑟𝑟
6
Step Four: check: 46 × 6 = 276

4) a. ½ tablet b. 160mg c. 1.3mL d. 60dpm e. 7.5mL

5) a. $140 b. $512 x 1.06 = $542.72 per week c. 25%


24 22
d. Score 1 = = 80%; Score 2 = = 73.3%. Fiona’s score decreased (80 – 73.3) 6.7%
30 30

e. 299−249 = 50 we work out the difference first, then we put the difference over
50 100
the original whole. × = 16.7%
299 1

6) What do I need to get on my final exam?

Credit (70.83%); Distinction (95.83%); HD (120.83 which is not achievable)

181

You might also like